Eco

Réussis tes devoirs et examens dès maintenant avec Quizwiz!

B

QN=128 (17248) In the market for oil in the short run, demand a. and supply are both elastic. b. and supply are both inelastic. c. is elastic and supply is inelastic. d. is inelastic and supply is elastic.

C

QN=280 (17422) Without government intervention, public goods tend to be a. overproduced and common resources tend to be overconsumed. b. overproduced and common resources tend to be underconsumed. c. underproduced and common resources tend to be overconsumed. d. underproduced and common resources tend to be underconsumed.

D

QN=281 (17405) What causes the Tragedy of the Commons? (i) Social and private incentives differ. (ii) Common resources are not rival in consumption and are not excludable. (iii) Common resources are not excludable but are rival in consumption. a. (i) only b. (ii) only c. (i) and (ii) only d. (i) and (iii) only

D

QN=282 (17407) A good is excludable if a. one person's use of the good diminishes another person's enjoyment of it. b. the government can regulate its availability. c. it is not a normal good. d. people can be prevented from using it.

B

QN=129 (17252) For a particular good, a 12 percent increase in price causes a 3 percent decrease in quantity demanded. Which of the following statements is most likely applicable to this good? a. There are many substitutes for this good. b. The good is a necessity. c. The market for the good is narrowly defined. d. The relevant time horizon is long.

A

QN=13 (17142) The term market failure refers to a. a situation in which the market on its own fails to allocate resources efficiently. b. an unsuccessful advertising campaign which reduces demand for a product. c. a situation in which competition among firms becomes ruthless. d. a firm which is forced out of business because of losses.

A

QN=130 (17227) The price elasticity of demand for a good measures the willingness of a. consumers to buy less of the good as price rises. b. consumers to avoid monopolistic markets in favor of competitive markets. c. firms to produce more of a good as price rises. d. firms to cater to the tastes of consumers.

C

QN=131 (17233) Some firms eventually experience problems with their capacity to produce output as their output levels increase. For these firms, a. market power is substantial. b. supply is perfectly inelastic. c. supply is more elastic at low levels of output and less elastic at high levels of output. d. supply is less elastic at low levels of output and more elastic at high levels of output.

A

QN=132 (17242) When the price of a good is $5, the quantity demanded is 120 units per month; when the price is $7, the quantity demanded is 100 units per month. Using the midpoint method, the price elasticity of demand is about a. 0.55. b. 1.83. c. 2. d. 0.45

A

QN=133 (17261) Refer to Figure 5-12. Over which range is the supply curve in this figure the most elastic? a. Between $16 and $40 b. Between $40 and $100 c. Between $100 and $220 d. Between $220 and $430

C

QN=283 (17394) Because public goods are a. excludable, people have an incentive to be free riders. b. excludable, people do not have an incentive to be free riders. c. not excludable, people have an incentive to be free riders. d. not excludable, people do not have an incentive to be free riders.

B

QN=191 (17303) Which of the following causes a shortage of a good? a. (i) a binding price floor b. (ii) a binding price ceiling c. (iii) a tax on the good d. More than one of (i), (ii), and (iii) is correct.

B

QN=300 (17409) An overcrowded beach is an example of a. a positive externality. b. a Tragedy of the Commons. c. an environmentally inefficient allocation of resources. d. an economically unfair allocation of resources.

D

QN=301 (17390) For most goods in an economy, the signal that guides the decisions of buyers and sellers is a. preference. b. government intervention. c. quantity. d. price.

B

QN=192 (17270) Price controls are usually enacted a. (i) as a means of raising revenue for public purposes. b. (ii) when policymakers believe that the market price of a good or service is unfair to buyers or sellers. c. (iii) when policymakers detect inefficiencies in a market. d. All of (i), (ii), and (iii) are correct.

C

QN=193 (17312) Ray buys a new tractor for $118,000. He receives consumer surplus of $13,000 on his purchase. Ray's willingness to pay is a. $13,000. b. $105,000. c. $118,000. d. $131,000.

C

QN=194 (17346) Refer to Figure 7-7. Which area represents the increase in producer surplus when the price rises from P1 to P2 due to new producers entering the market? a. BCG b. ACH c. DGH d. AHGB

D

QN=302 (17412) Which of the following would not be considered a private good? a. a pair of jeans b. an apple c. a Honda Civic d. cable TV service

C

QN=195 (17317) Market power refers to the a. side effects that may occur in a market. b. government regulations imposed on the sellers in a market. c. ability of market participants to influence price. d. forces of supply and demand in determining equilibrium price.

D

QN=303 (17425) The sign on a church in your neighborhood reads "All are welcome at Sunday Service." Because the church has limited seating and is usually full, the Sunday Service is a. a private good. b. a public good. c. a natural monopoly. d. a common resource.

D

QN=325 (17440) Industrial organization is the study of how a. labor unions organize workers in industries. b. profitable firms are in organized industries. c. industries organize for political advantage. d. firms' decisions regarding prices and quantities depend on the market conditions they face.

A

QN=326 (17453) The long-run average total cost curve is always a. flatter than the short-run average total cost curve, but not necessarily horizontal. b. horizontal. c. falling as output increases. d. rising as output increases.

B

QN=327 (17449) At Bert's Bootery, the total cost of producing twenty pairs of boots is $400. The marginal cost of producing the twenty-first pair of boots is $83. We can conclude that the a. average variable cost of 21 pairs of boots is $23. b. average total cost of 21 pairs of boots is $23. c. average total cost of 21 pairs of boots is $15.09. d. marginal cost of the 20th pair of boots is $20.

A

QN=328 (17457) Implicit costs a. do not require an outlay of money by the firm. b. do not enter into the economist's measurement of a firm's profit. c. are also known as variable costs. d. are not part of an economist's measurement of opportunity cost.

B

QN=329 (17469) If a firm experiences constant returns to scale at all output levels, then its long-run average total cost curve would a. slope downward. b. be horizontal. c. slope upward. d. slope downward for low output levels and upward for high output levels.

D

QN=33 (17119) Trade between countries tends to a. reduce both competition and specialization. b. reduce competition and increase specialization. c. increase competition and reduce specialization. d. increase both competition and specialization.

D

QN=330 (17464) If a firm produces nothing, which of the following costs will be zero? a. total cost b. fixed cost c. opportunity cost d. variable cost

C

QN=350 (17451) Refer to Table 13-1. Alyson's pet sitting service experiences diminishing marginal productivity with the addition of the a. first worker. b. second worker. c. third worker. d. fourth worker.

B

QN=351 (17436) Which field of economics studies how the number of firms affects the prices in a market and the efficiency of market outcomes? a. macro economics b. industrial organization c. labor economics d. monetary economics

D

QN=352 (17442) The curves below reflect information about the cost structure of a firm. Use the figure to answer the following questions Refer to Figure 13-5. Curve A is U-shaped because of a. diminishing marginal product. b. increasing marginal product. c. the fact that increasing marginal product follows decreasing marginal product. d. the fact that decreasing marginal product follows increasing marginal product.

C

QN=104 (17223) Which of the following would shift the supply curve for gasoline to the right? a. An increase in the demand for gasoline. b. An increase in the price of gasoline. c. An increase in the number of producers of gasoline d. An increase in the price of oil, an input into the production of gasoline.

A

QN=105 (17212) A supply schedule is a table that shows the relationship between a. price and quantity supplied. b. input costs and quantity supplied. c. quantity demanded and quantity supplied. d. profit and quantity supplied.

D

QN=106 (17188) Soup is an inferior good if a. The demand for soup falls when the price of a substitute for soup rises. b. The demand for soup rises when the price of soup falls. c. The demand curve for soup slopes upward. d. The demand for soup falls when income rises.

D

QN=107 (17211) Which of the following is an example of a market? a. (i) a gas station b. (ii) a garage sale c. (iii) a barber shop d. All of (i), (ii), and (iii) are examples of markets.

A

QN=215 (17338) Which of the following events would increase producer surplus? a. (i) Sellers' costs stay the same and the price of the good increases. b. (ii) Sellers' costs increase and the price of the good stays the same. c. (iii) Sellers' costs increase and the price of the good decreases. d. All of (i), (ii), and (iii) are correct.

D

QN=108 (17187) In a competitive market, the price of a product a. (i) is determined by buyers and the quantity of the product produced is determined by sellers. b. (ii) is determined by sellers and the quantity of the product produced is determined by buyers. c. (iii) And the quantity of the product produced are both determined by sellers. d. None of (i), (ii), and (iii) is correct.

C

QN=216 (17316) Refer to Figure 7-1. If the supply curve is S, the demand curve is D, and the equilibrium price is $100, what is the producer surplus? a. $625 b. $1,250 c. $2,500 d. $5,000

B

QN=217 (17323) Suppose there is an early freeze in California that ruins the lemon crop. What happens to consumer surplus in the market for lemons? a. It increases. b. It decreases. c. It is not affected by this change in market forces. d. It increases very briefly then decreases.

A

QN=218 (17339) Consumer surplus equals the a. value to buyers minus the amount paid by buyers. b. value to buyers minus the cost to sellers. c. amount received by sellers minus the cost to sellers. d. amount received by sellers minus the amount paid by buyers.

B

QN=219 (17331) 2. Refer to Figure 7-15. If the price decreases from $22 to $16 due to a shift in the supply curve, consumer surplus increases by a. $120. b. $360. c. $480. d. $600.

C

QN=22 (17129) A tradeoff exists between a clean environment and a higher level of income in that a. studies show that individuals with higher levels of income actually pollute less than low-income individuals. b. to pay for pollution clean-up, the government must increase taxes which lowers income. c. laws that reduce pollution raise costs of production and reduce incomes. d. by employing individuals to clean up pollution, employment and income both rise.

C

QN=125 (17263) Scenario 5-1 The supply of aged cheddar cheese is inelastic and the supply of bread is elastic. Both goods are considered to be normal goods by a majority of consumers. Suppose that a large income tax increase decreases the demand for both goods by 10%. Refer to Scenario 5-1. The price elasticity of supply for aged cheddar cheese could be a. -1. b. 0. c. 0.5. d. 1.5.

D

QN=220 (17337) Refer to Figure 7-2. When the price rises from P1 to P2, consumer surplus a. increases by an amount equal to A. b. decreases by an amount equal to B+C. c. increases by an amount equal to B+C. d. decreases by an amount equal to C.

A

QN=126 (17245) The price elasticity of demand measures a. buyers' responsiveness to a change in the price of a good. b. the extent to which demand increases as additional buyers enter the market. c. how much more of a good consumers will demand when incomes rise. d. the movement along a supply curve when there is a change in demand.

A

QN=259 (17359) The demand curve for a product reflects the a. value of the product to consumers. b. cost of the product to consumers. c. quantity consumers are able to purchase. d. price the product will sell for in the market.

D

QN=26 (17137) The tradeoff between inflation and unemployment a. (i) implies that policies designed to reduce unemployment also reduce inflation. b. (ii) was eliminated by improved economic policies in the 1900s. c. (iii) is a long-run tradeoff, persisting for decades, according to most economists. d. None of (i), (ii), and (iii) are correct.

D

QN=260 (17376) Which of the following statements is not correct? a. Government policies may improve the market's allocation of resources when negative externalities are present. b. Government policies may improve the market's allocation of resources when positive externalities are present. c. A positive externality is an example of a market failure. d. Without government intervention, the market will tend to undersupply products that produce negative externalities.

A

QN=261 (17362) Refer to Figure 10-4. At Q3 a. the marginal consumer values this product less than the social cost of producing it. b. every consumer values this product less than the social cost of producing it. c. the cost to society is equal to the value to society. d. the marginal consumer values this product more than the private cost.

A

QN=127 (17236) An inelastic demand means that a. consumers hardly respond to a change in price. b. consumers respond substantially to a change in price. c. consumers respond directly to a change in income. d. the change in quantity demanded is equal to the change in price.

D

QN=477 (17564) Refer to Table 16-3. Which of the following is likely to happen in the long run in this market? a. The market is currently in a long-run equilibrium. b. The market price is likely to fall. c. Firms are likely to enter the market since firms are earning a positive economic profit. d. Firms are likely to leave the market since firms are earning a negative economic profit.

C

QN=478 (17593) Which of the following statements is correct? a. Monopolistic competition is similar to monopoly because both market structures are characterized by patents. b. Monopolistic competition is similar to perfect competition because both market structures are characterized by each seller being small compared to the market. c. Monopolistic competition is similar to oligopoly because both market structures are characterized by free entry. d. Monopolistic competition is similar to perfect competition because both market structures are characterized by excess capacity.

C

QN=479 (17595) Refer to Figure 16-3. The firm in this figure is monopolistically competitive. It illustrates a. the shut-down case. b. a long-run economic profit. c. a short-run economic profit. d. a short-run loss.

A

QN=48 (17158) When economists attempt to simplify the real world and make it easier to understand they make a. assumptions. b. mistakes in judgment. c. predictions. d. evaluations.

D

QN=480 (17583) Monopolistic competition is characterized by which of the following attributes? (i) free entry (ii) product differentiation (iii) many sellers a. (i) and (iii) only b. (i) and (ii) only c. (ii) and (iii) only d. (i), (ii), and (iii)

C

QN=481 (17571) Cartels are difficult to maintain because a. (i) antitrust laws are difficult to enforce. b. (ii) cartel agreements are conducive to monopoly outcomes. c. (iii) there is always tension between cooperation and self-interest in a cartel. d. All of (i), (ii), and (iii) are correct.

C

QN=482 (17587) Which of the following conditions is characteristic of a monopolistically competitive firm in short-run equilibrium? a. (i) P > AR b. (ii) MR > MC c. (iii) P > MC d. All of (i), (ii), and (iii) are correct.

A

QN=483 (17585) A profit-maximizing firm in a monopolistically competitive market is characterized by which of the following? a. average revenue exceeds marginal revenue b. marginal revenue exceeds average revenue c. average revenue is equal to marginal revenue d. revenue is always maximized along with profit

C

QN=484 (17578) A monopolistically competitive market has characteristics that are similar to a. a monopoly only. b. a competitive firm only. c. both a monopoly and a competitive firm. d. neither a monopoly nor a competitive firm.

B

QN=485 (17580) A monopolistically competitive firm faces the following demand schedule for its product: The firm has total fixed costs of $20 and a constant marginal cost of $2 per unit. The firm will maximize profit with a. 6 units of output. b. 9 units of output. c. 11 units of output. d. 13 units of output.

B

QN=486 (17586) To maximize its profit, a monopolistically competitive firm chooses its level of output by looking for the level of output at which a. (i) price equals marginal cost. b. (ii) marginal revenue equals marginal cost. c. (iii) average total cost is minimized. d. All of (i), (ii), and (iii) are correct.

A

QN=487 (17591) Entry by new firms into a monopolistically competitive market a. creates additional consumer surplus. b. imposes a positive externality on existing firms. c. leads to the same externalities that are observed when new firms enter a perfectly competitive market. d. increases the demand for existing firms' products.

D

QN=592 (17683) When a consumer experiences a price decrease for an inferior good, it is possible that the income effect is a. (i) less than the substitution effect, and the demand curve will be downward sloping. b. (ii) greater than the substitution effect, and the demand curve will be upward sloping. c. (iii) less than the substitution effect, and the demand curve will be upward sloping. d. both (i) and (ii) are correct.

A

QN=571 (17687) Just as the theory of the competitive firm provides a more complete understanding of supply, the theory of consumer choice provides a more complete understanding of a. demand. b. profits. c. production possibility frontiers. d. wages.

B

QN=572 (17710) If the price of a good is low, a. firms would increase profit by increasing output. b. the quantity supplied of the good could be zero. c. the supply curve for the good will shift to the left. d. firms can and should raise the price of the product.

A

QN=573 (17709) Refer to Figure 21-4. In graph (b), what is the price of good X relative to good Y (i.e., Px/Py)? a. 2/7 b. 3/6 c. 7/2 d. 7

C

QN=593 (17702) Refer to Figure 21-2. Which points are affordable? a. W, X, and Y only b. Z only c. V, W, X, and Y only d. V, W, X, Y, and Z

C

QN=594 (17708) 3. Refer to Figure 21-4. In graph (a), if income is equal to $120, the price of good Y is a. $1 b. $2 c. $3 d. $4

D

QN=574 (17689) Which of the following could explain the change in the budget line from A to B? a. (i) a simultaneous decrease in the price of X and the price of Y b. (ii) an increase in income c. (iii) an increase in income and a decrease in the price of Y d. Both (i) and (ii) are correct.

D

QN=575 (17706) 1. Refer to Figure 21-2. A consumer that chooses to spend all of her income could be at which point(s) on the budget constraint? a. V only b. Z only c. V, W, X, or Y only d. W, X, or Y only

B

QN=591 (17695) The theory of consumer choice can often provide insight into the behavior of a. individuals who make rational choices. b. individuals who make constrained choices. c. individuals who are unaware of how to maximize their well-being. d. irrational consumers.

A

QN=595 (17691) If the relative price of a ticket to a concert is 3 times the price of a meal at a good restaurant, the opportunity cost of a concert ticket is the a. slope of the budget constraint. b. slope of the indifference curve. c. intercept on the concert axis. d. intercept on the restaurant axis.

D

QN=596 (17684) Diana and Sarah each like jewelry and music by the Rolling Stones. If we were to graph an indifference curve with jewelry on the horizontal axis and cd's by the Rolling Stones on the vertical axis, then a. Diana and Sarah would have identical indifference curves. b. Diana's indifference curve would be higher than Sarah's indifference curve. c. Sarah's indifference curve would be higher than Diana's indifference curve. d. Because we do not know the intensity of each woman's preferences, we do not have enough information to compare their indifference curves.

D

QN=597 (17716) When a consumer experiences a price decrease for an inferior good, it is possible that the income effect is a. (i) less than the substitution effect, and the demand curve will be downward sloping. b. (ii) greater than the substitution effect, and the demand curve will be upward sloping. c. (iii) less than the substitution effect, and the demand curve will be upward sloping. d. both (i) and (ii) are correct.

C

QN=598 (17690) If the price of X is $10, what is the price of Y? a. $15 b. $25 c. $35 d. $70

D

QN=72 (17177) Factors of production are a. the mathematical calculations firms make in determining their optimal production levels. b. social and political conditions that affect production. c. the physical relationships between economic inputs and outputs. d. inputs into the production process.

B

QN=73 (17182) Which of the following is not an example of a positive, as opposed to normative, statement? a. Higher gasoline prices will reduce gasoline consumption. b. Equality is more important than efficiency. c. Trade restrictions lower our standard of living. d. If a nation wants to avoid

B

QN=74 (17204) Two goods are substitutes if a decrease in the price of one good a. increases the demand for the other good. b. reduces the demand for the other good. c. reduces the quantity demanded of the other good. d. increases the quantity demanded of the other good.

D

QN=75 (17209) Who gets scarce resources in a market economy? a. the government b. whoever the government decides gets them c. whoever wants them d. whoever is willing and able to pay the price

C

QN=76 (17220) Refer to Table 4-2. Whose demand does not obey the law of demand? a. Audrey's b. Bob's c. Chuck's d. Dottie's

D

QN=77 (17205) In a market economy, supply and demand are important because they a. (i) play a critical role in the allocation of the economy's scarce resources. b. (ii) determine how much of each good gets produced. c. (iii) can be used to predict the impact on the economy of various events and policies. d. All of (i), (ii), and (iii) are correct.

B

QN=78 (17197) Which of the following statements is correct? a. Buyers determine supply and sellers determine demand. b. Buyers determine demand and sellers determine supply. c. Buyers determine both demand and supply. d. Sellers determine both demand and supply.

D

QN=79 (17224) What will happen to the equilibrium price of new textbooks if more students attend college, paper becomes cheaper, textbook authors accept lower royalties, and fewer used textbooks are sold? a. Price will rise. b. Price will fall. c. Price will stay exactly the same. d. The price change will be ambiguous.

B

QN=8 (17147) The invisible hand refers to a. how central planners made economic decisions. b. how the decisions of households and firms lead to desirable market outcomes. c. the control that large firms have over the economy. d. government regulations without which the economy would be less efficient.

B

QN=80 (17206) Suppose that Carolyn receives a pay increase. We would expect a. to observe Carolyn moving down and to the right along her given demand curve. b. Carolyn's demand for inferior goods to decrease. c. Carolyn will go shopping at a clothing store worse than before. d. Carolyn's demand for normal goods to decrease.

B

QN=81 (17217) If the price of a good is low, a. firms would increase profit by increasing output. b. the quantity supplied of the good could be zero. c. the supply curve for the good will shift to the left. d. firms can and should raise the price of the product.

B

QN=82 (17216) Today's demand curve for gasoline could shift in response to a. (i) a change in today's price of gasoline. b. (ii) a change in the expected future price of gasoline. c. (iii) a change in the number of sellers of gasoline. d. All of (i), (ii), and (iii) are correct.

B

QN=353 (17429) The minimum points of the average variable cost and average total cost curves occur where a. the marginal cost curve lies below the average variable cost and average total cost curves. b. the marginal cost curve intersects those curves. c. the average variable cost and average total cost curves intersect. d. the slope of total cost is the smallest.

C

QN=354 (17455) A firm's opportunity costs of production are equal to its a. explicit costs only. b. implicit costs only. c. explicit costs + implicit costs. d. explicit costs + implicit costs + total revenue.

C

QN=355 (17452) Suppose Jan started up a small lemonade stand business last month. Variable costs for Jan's lemonade stand now include the cost of a. (i) building the lemonade stand. b. (ii) hiring an artist to design a logo for her sign. c. (iii) lemons and sugar. d. All of (i), (ii), and (iii) are correct.

C

QN=356 (17466) Refer to Table 13-6. What is the average variable cost of producing 5 units of output? a. $4 b. $5 c. $40 d. $44

D

QN=357 (17447) The marginal cost curve crosses the average total cost curve at a. (i) the efficient scale. b. (ii) the minimum point on the average total cost curve. c. (iii) a point where the marginal cost curve is rising. d. All of (i), (ii), and (iii) are correct.

C

QN=358 (17495) Which of the following is not a characteristic of a perfectly competitive market? a. Firms are price takers. b. Firms can freely enter the market. c. Many firms have market power. d. Goods offered for sale are largely the same.

A

QN=359 (17509) In a competitive market, the actions of any single buyer or seller will a. have a negligible impact on the market price. b. have little effect on market equilibrium quantity but will affect market equilibrium price. c. affect marginal revenue and average revenue but not price. d. adversely affect the profitability of more than one firm in the market.

A

QN=36 (17173) Another term for factors of production is a. inputs. b. output. c. goods. d. services.

C

QN=360 (17479) For any competitive market, the supply curve is closely related to the a. preferences of consumers who purchase products in that market. b. income tax rates of consumers in that market. c. firms' costs of production in that market. d. interest rates on government bonds.

B

QN=361 (17492) As a general rule, when accountants calculate profit they account for explicit costs but usually ignore a. certain outlays of money by the firm. b. implicit costs. c. operating costs. d. fixed costs.

C

QN=362 (17515) When fixed costs are ignored because they are irrelevant to a business's production decision, they are called a. explicit costs. b. implicit costs. c. sunk costs. d. opportunity costs.

B

QN=363 (17517) Refer to Figure 14-9. If the market starts in equilibrium at point C in panel (b), a decrease in demand will ultimately lead to a. more firms in the industry but lower levels of output for each firm. b. fewer firms in the market. c. a new long-run equilibrium at point D in panel (b). d. lower prices once the new long-run equilibrium is reached.

A

QN=383 (17488) The assumption of a fixed number of firms is appropriate for analysis of a. the short run, but not the long run. b. the long run, but not the short run. c. both the short run and the long run. d. neither the short run nor the long run.

A

QN=384 (17499) For a firm in a perfectly competitive market, the price of the good is always a. equal to marginal revenue. b. equal to total revenue. c. greater than average revenue. d. equal to the firm's efficient scale of output.

C

QN=385 (17501) When a profit-maximizing firm is earning profits, those profits can be identified by a. P*Q. b. (MC - AVC)*Q. c. (P - ATC)*Q. d. (P - AVC)*Q.

D

QN=402 (17500) Suppose a firm operates in the short run at a price above its average total cost of production. In the long run the firm should expect a. (i) new firms to enter the market. b. (ii) the market price to fall. c. (iii) its profits to fall. d. All of (i), (ii), and (iii) are correct.

D

QN=403 (17476) A competitive firm would benefit from charging a price below the market price because the firm would achieve a. (i) higher average revenue. b. (ii) higher profits. c. (iii) lower total costs. d. None of (i), (ii), and (iii) is correct.

D

QN=404 (17493) 5. Refer to Table 14-5. The maximum profit available to this firm is a. $2. b. $3. c. $4. d. $5.

C

QN=405 (17505) Suppose that a firm operating in a perfectly competitive market sells 400 units of output at a price of $4 each. Which of the following statements is correct? (i) Marginal revenue equals $4. (ii) Average revenue equals $100. (iii) Total revenue equals $1,600. a. (i) only b. (iii) only c. (i) and (iii) only d. (i), (ii), and (iii)

A

QN=406 (17484) When a firm has little ability to influence market prices it is said to be in what kind of a market? a. a competitive market b. a strategic market c. a thin market d. a power market

B

QN=407 (17557) For a monopolist, when the price effect is greater than the output effect, marginal revenue is a. positive. b. negative. c. zero. d. maximized.

B

QN=408 (17542) Refer to Table 15-5. If the monopolist faces a constant marginal cost of $2, how much output should the firm produce? a. 3 units b. 4 units c. 5 units d. 6 units

B

QN=409 (17555) A natural monopoly occurs when a. the product is sold in its natural state, such as water or diamonds. b. there are economies of scale over the relevant range of output. c. the firm is characterized by a rising marginal cost curve. d. production requires the use of free natural resources, such as water or air.

A

QN=41 (17160) Refer to Figure 2-1. Which arrow shows the flow of spending by households? a. A b. B c. C d. D

D

QN=410 (17553) When a monopolist is able to sell its product at different prices, it is engaging in a. distribution pricing. b. quality-adjusted pricing. c. price differentiation. d. price discrimination.

A

QN=411 (17538) If one were to compare a competitive market to a monopoly that engages in perfect price discrimination, one could say that a. in both cases, total social welfare is the same. b. total social welfare is maximized in the competitive market, but not in the perfectly discriminating monopoly. c. in both cases, some potentially mutually beneficial trades do not occur. d. consumer surplus is the same in both cases.

B

QN=412 (17540) Name brand drugs are able to continue capitalizing on their market power even after generic drugs enter the market because (i) almost all people fear the generic drug companies are devoting too few resources to research and development. (ii) some people fear that generic drugs are inferior. (iii) some people are loyal to the name brand. a. (i) and (ii) only b. (ii) and (iii) only c. (i) and (iii) only d. (i), (ii), and (iii)

B

QN=413 (17556) What is the shape of the monopolist's marginal revenue curve? a. a downward-sloping line that is identical to the demand curve b. a downward-sloping line that lies below the demand curve c. a horizontal line that is identical to the demand curve d. a horizontal line that lies below the demand curve

B

QN=414 (17552) Price discrimination is the business practice of a. bundling related products to increase total sales. b. selling the same good at different prices to different customers. c. pricing above marginal cost. d. hiring marketing experts to increase consumers' brand loyalty.

C

QN=415 (17551) Refer to Figure 15-3. What area measures the monopolist's profit? a. (B-F)*K b. (A-H)*J c. (B-G)*K d. 0.5[(B-F)*(L-K)]

A

QN=416 (17539) If a monopolist sells 100 units at $8 per unit and realizes an average total cost of $6 per unit, what is the monopolist's profit? a. $200 b. $400 c. $600 d. $800

B

QN=417 (17544) Refer to Figure 15-1. Considering the relationship between average total cost and marginal cost, the marginal cost curve for this firm a. must lie entirely above the average total cost curve. b. must lie entirely below the average total cost curve. c. must be upward sloping. d. does not exist.

C

QN=455 (17598) Critics of advertising argue that in some markets advertising may a. attract products of lower quality into the market. b. attract less informed buyers into the market. c. decrease elasticity of demand allowing firms to charge a larger markup over marginal cost. d. enhance competition in markets to an unnecessary degree.

C

QN=456 (17589) In a monopolistically competitive industry, a firm's demand curve also represent its a. marginal revenue. b. marginal cost. c. average revenue. d. profit.

D

QN=476 (17563) A profit-maximizing firm in a monopolistically competitive market is characterized by which of the following? a. (i) average revenue exceeds marginal revenue b. (ii) marginal revenue equals marginal cost c. (iii) price exceeds marginal cost d. All of (i), (ii), and (iii) are correct.

B

If the quantity demanded of a certain good responds only slightly to a change in the price of the good, then a. the demand for the good is said to be elastic. b. the demand for the good is said to be inelastic. c. the law of demand does not apply to the good. d. the demand curve for the good shifts only slightly in response to a change in price.

C

QN=24 (17115) In a market economy, who makes the decisions that guide most economic activities? a. firms only b. households only c. firms and households d. government

A

QN=1 (17143) The marginal benefit John gets from eating a fourth cheeseburger at a picnic is a. the total benefit John gets from eating four cheeseburgers minus the total benefit John gets from eating three cheeseburgers. b. the same as the total benefit of eating four cheeseburgers. c. less than the marginal cost of eating the fourth cheeseburger since he chose to eat the fourth cheeseburger. d. the total benefit John gets from eating five cheeseburgers minus the total benefit John gets from eating four cheeseburgers.

D

QN=10 (17138) Which of the following statements best represents the principle represented by the adage, "There is no such thing as a free lunch"? a. Melissa can attend the concert only if she takes her sister with her. b. Greg is hungry and homeless. c. Brian must repair the tire on his bike before he can ride it to class. d. Kendra must decide between going to Colorado or Cancun for spring break.

B

QN=100 (17202) Generally, the market for ice cream would be considered a. a monopolistic market. b. a competitive market. c. more organized than an auction. d. a market where individual sellers have significant pricing power.

A

QN=101 (17203) If buyers and/or sellers are price takers, then individually a. they have no influence on market price. b. they have ultimate control over market price. c. buyers will be able to find prices lower than those determined in the market. d. they can somewhat influence the market price.

C

QN=102 (17208) 4. Refer to Table 4-7. Suppose Charlie, Maxine, and Quinn are the only demanders of sandwiches and that the market demand violates the law of demand. Then, in the table, a. x 5. b. x 5. c. x 7. d. x 10.

C

QN=103 (17207) 3. Refer to Figure 4-4. The graphs show the demand for cigarettes. In Panel (b), the leftward arrow is consistent with which of the following events? a. The price of cigarettes increased. b. A sales tax was placed on cigarettes. c. The prohibition of cigarette advertisements on television. d. Tobacco and marijuana are complements and the price of marijuana decreased.

A

QN=109 (17195) 5. Refer to Table 4-7. Suppose Charlie, Maxine, and Quinn are the only demanders of sandwiches. Also suppose the following: • x = 2 • the current price of a sandwich is $3.00 • the market quantity supplied of sandwiches is 5 • the slope of the supply curve is 1 Then a. there is currently a shortage of 5 sandwiches and the equilibrium price of a sandwich is between $3.00 and $5.00. b. there is currently a shortage of 5 sandwiches and the equilibrium price of a sandwich is $5.00. c. there is currently a surplus of 5 sandwiches and the equilibrium price of a sandwich is between $3.00 and $5.00. d. there is currently a surplus of 5 sandwiches and the equilibrium price of a sandwich is $5.00.

B

QN=11 (17145) Suppose a typical worker in France can produce 32 units of product in an eight-hour day, while a typical worker in Germany can produce 30 units of product in a 10-hour day. We can conclude that a. worker productivity in Germany is higher than in France. b. the standard of living will likely be higher in France than in Germany. c. productivity is 4 units per hour for the German worker and 3 units per hour for the French worker. d. there will be no difference between the standard of living in France and Germany.

D

QN=110 (17201) A competitive market is one in which a. there is only one seller of the product. b. each seller of the product is free to set the price of his product. c. each seller attempts to compete with other sellers, causing fewer sellers in the market. d. there are so many buyers and many sellers that each has a negligible impact on price.

A

QN=111 (17225) Pens are normal goods. What will happen to the equilibrium price of pens if the price of pencils rises, consumers experience an increase in income, writing in ink becomes fashionable, people expect the price of pens to rise in the near future, the population increases, fewer firms manufacture pens, and the wages of pen-makers increase? a. Price will rise. b. Price will fall. c. Price will stay exactly the same. d. The price change will be ambiguous.

A

QN=112 (17221) Which of the following would not shift the demand curve for mp3 players? a. a decrease in the price of mp3 players b. a trend that makes mp3 players more popular among 12-25 year olds c. an increase in the price of CDs, a complement for mp3 players d. a decrease in the price of satellite radio, a substitute for mp3 players

C

QN=113 (17219) Which of the following is not held constant in a demand schedule? a. income b. tastes c. price d. expectations

A

QN=114 (17243) The price elasticity of supply measures how responsive a. sellers are to a change in price. b. sellers are to a change in buyers' income. c. buyers are to a change in production costs. d. equilibrium price is to a change in supply.

A

QN=115 (17230) A decrease in supply, which is a leftward shift of the supply curve, will cause the largest increase in price when a. both supply and demand are inelastic. b. both supply and demand are elastic. c. demand is elastic and supply is inelastic. d. demand is inelastic and supply is elastic.

A

QN=116 (17239) The flatter the demand curve through a given point, the a. greater the price elasticity of demand. b. smaller the price elasticity of demand. c. closer the price elasticity of demand will be to the slope of the curve. d. more equal the price elasticity of demand will be to the slope of the curve.

A

QN=117 (17238) Most economists report the elasticity of demand as a. the absolute value of the actual number. b. a negative number, since price and quantity demanded move in opposite directions. c. a percentage, since both the numerator and denominator are percentages. d. a dollar amount, since we are measuring the change in price.

C

QN=118 (17260) If the price elasticity of supply is 1.5, and a price increase led to a 1.8% increase in quantity supplied, then the price increase amounted to a. 0.67%. b. 0.83%. c. 1.20%. d. 2.70%.

A

QN=119 (17258) Income elasticity of demand measures how a. the quantity demanded changes as consumer income changes. b. consumer purchasing power is affected by a change in the price of a good. c. the price of a good is affected when there is a change in consumer income. d. many units of a good a consumer can buy given a certain income level.

C

QN=12 (17146) Russell spends an hour studying instead of playing tennis. The opportunity cost to him of studying is a. the improvement in his grades from studying for the hour. b. the improvement in his grades from studying minus the enjoyment of playing tennis. c. the enjoyment and exercise he would have received had he played tennis. d. zero. Since Russell chose to study rather than to play tennis, the value of studying must have been greater than the value of playing tennis.

B

QN=120 (17257) If the demand for textbooks is inelastic, then a decrease in the price of textbooks will a. increase total revenue of textbook sellers. b. decrease total revenue of textbook sellers. c. not change total revenue of textbook sellers. d. There is not enough information to answer this question.

A

QN=121 (17234) To determine whether a good is considered normal or inferior, one could examine the value of the a. income elasticity of demand for that good. b. price elasticity of demand for that good. c. price elasticity of supply for that good. d. cross-price elasticity of demand for that good.

C

QN=122 (17262) Refer to Figure 5-12. Using the midpoint method, what is the price elasticity of supply between $16 and $40? a. 0.125 b. 0.86 c. 1.0 d. 2.5

A

QN=123 (17264) Scenario 5-2 The supply of aged cheddar cheese is inelastic, and the supply of bread is elastic. Both goods are considered to be normal goods by a majority of consumers. Suppose that a large income tax increase decreases the demand for both goods by 10%. Refer to Scenario 5-2. The change in equilibrium price will be a. greater in the aged cheddar cheese market than in the bread market. b. greater in the bread market than in the aged cheddar cheese market. c. the same in the aged cheddar cheese and bread markets. d. may be greater in either the aged cheddar cheese market or the bread market.

C

QN=124 (17244) Which of the following statements helps to explain why government drug interdiction increases drug-related crime? a. The direct impact is on buyers, not sellers. b. Successful drug interdiction policies reduce the demand for illegal drugs. c. Drug addicts will have an even greater need for quick cash to support their habits. d. In the short run, both equilibrium quantities and prices will fall in the markets for illegal drugs.

B

QN=134 (17259) Which of the following expressions represents a cross-price elasticity of demand? a. percentage change in quantity demanded of bread divided by percentage change in quantity supplied of bread b. percentage change in quantity demanded of bread divided by percentage change in price of butter c. percentage change in price of bread divided by percentage change in quantity demanded of bread d. percentage change in quantity demanded of bread divided by percentage change in income

B

QN=135 (17237) If a good is a luxury, demand for the good would tend to be a. inelastic. b. elastic. c. unit elastic. d. horizontal.

C

QN=136 (17250) Which of the following statements is correct? a. (i) The demand for natural gas is more elastic over a short period of time than over a long period of time. b. (ii) The demand for smoke alarms is more elastic than the demand for Persian rugs. c. (iii) The demand for bourbon whiskey is more elastic than the demand for alcoholic beverages in general. d. All of (i), (ii), and (iii) are correct.

B

QN=137 (17240) Suppose there is a 6 percent increase in the price of good X and a resulting 6 percent decrease in the quantity of X demanded. Price elasticity of demand for X is a. 0. b. 1. c. 6. d. 36.

B

QN=138 (17251) Which of the following is not a determinant of the price elasticity of demand for a good? a. the time horizon b. the steepness or flatness of the supply curve for the good c. the definition of the market for the good d. the availability of substitutes for the good

D

QN=139 (17256) Refer to Table 5-3. Using the midpoint method, what is the price elasticity of demand when price rises from $9 to $12? a. 0.43 b. 0.67 c. 1.50 d. 2.33

D

QN=14 (17117) In considering how to allocate its scarce resources among its various members, a household considers a. (i) each member's abilities. b. (ii) each member's efforts. c. (iii) each member's desires. d. all of (i), (ii), and (iii).

C

QN=140 (17228) If the price elasticity of demand for a good is 0.4, then a 10 percent increase in price results in a a. 0.4 percent decrease in the quantity demanded. b. 2.5 percent decrease in the quantity demanded. c. 4 percent decrease in the quantity demanded. d. 40 percent decrease in the quantity demanded.

D

QN=141 (17265) Scenario 5-2 The supply of aged cheddar cheese is inelastic, and the supply of bread is elastic. Both goods are considered to be normal goods by a majority of consumers. Suppose that a large income tax increase decreases the demand for both goods by 10%. Refer to Scenario 5-2. The equilibrium price will a. increase in the aged cheddar cheese market and increase in the bread market. b. increase in the aged cheddar cheese market and decrease in the bread market. c. decrease in the aged cheddar cheese market and increase in the bread market. d. decrease in the aged cheddar cheese market and decrease in the bread market.

A

QN=142 (17226) When consumers face rising gasoline prices, they typically a. reduce their quantity demanded more in the long run than in the short run. b. reduce their quantity demanded more in the short run than in the long run. c. do not reduce their quantity demanded in the short run or the long run. d. increase their quantity demanded in the short run but reduce their quantity demanded in the long run.

B

QN=143 (17254) If the quantity demanded of a certain good responds only slightly to a change in the price of the good, then a. the demand for the good is said to be elastic. b. the demand for the good is said to be inelastic. c. the law of demand does not apply to the good. d. the demand curve for the good shifts only slightly in response to a change in price.

C

QN=144 (17232) A key determinant of the price elasticity of supply is the a. number of close substitutes for the good in question. b. definition of the market. c. length of the time period. d. extent to which buyers alter their quantities demanded in response to changes in their incomes.

A

QN=145 (17241) The flatter the demand curve through a given point, the a. greater the price elasticity of demand at that point. b. smaller the price elasticity of demand at that point. c. closer the price elasticity of demand will be to the slope of the curve. d. greater the absolute value of the change in total revenue when there is a movement from that point upward and to the left along the demand curve.

D

QN=147 (17247) Which of the following statements is not valid when supply is perfectly elastic? a. The elasticity of supply approaches infinity. b. The supply curve is horizontal. c. Very small changes in price lead to large changes in quantity supplied. d. The time period under consideration is more likely a short period rather than a long period.

C

QN=148 (17253) Demand is said to have unit elasticity if elasticity is a. less than 1. b. greater than 1. c. equal to 1. d. equal to 0.

A

QN=149 (17235) The most basic tools of economics are a. demand and supply. b. price and quantity. c. monetary and fiscal policy. d. elasticity of demand and supply.

B

QN=15 (17134) The term used to describe a situation in which markets do not allocate resources efficiently is a. economic meltdown. b. market failure. c. equilibrium. d. the effect of the invisible hand.

A

QN=150 (17231) Elasticity is a. a measure of how much buyers and sellers respond to changes in market conditions. b. the study of how the allocation of resources affects economic well-being. c. the maximum amount that a buyer will pay for a good. d. the value of everything a seller must give up to produce a good.

D

QN=151 (17246) Which of the following statements is correct? a. (i) The demand for flat-screen computer monitors is more elastic than the demand for monitors in general. b. (ii) The demand for grandfather clocks is more elastic than the demand for clocks in general. c. (iii) The demand for cardboard is more elastic over a long period of time than over a short period of time. d. All of (i), (ii), and (iii) are correct.

C

QN=152 (17229) Knowing that the demand for wheat is inelastic, if all farmers voluntarily did not plant wheat on 10 percent of their land, then a. consumers of wheat would buy more wheat. b. wheat farmers would suffer a reduction in their total revenue. c. wheat farmers would experience an increase in their total revenue. d. the demand for wheat would decrease.

D

QN=153 (17255) The price elasticity of demand for bread a. (i) is computed as the percentage change in quantity demanded of bread divided by the percentage change in price of bread. b. (ii) depends, in part, on the availability of close substitutes for bread. c. (iii) reflects the many economic, social, and psychological forces that influence consumers' tastes for bread. d. All of (i), (ii), and (iii) are correct.

C

QN=154 (17285) Refer to Table 6-1. Suppose the government imposes a price ceiling of $1 on this market. What will be the size of the shortage in this market? a. 0 units b. 2 units c. 8 units d. 10 units

C

QN=155 (17282) A price ceiling will only be binding if it is set a. equal to equilibrium price. b. above equilibrium price. c. below equilibrium price. d. A price ceiling is never binding in a free market system.

B

QN=157 (17276) Suppose the government has imposed a price ceiling on televisions. Which of the following events could transform the price ceiling from one that is not binding into one that is binding? a. Firms expect the price of televisions to fall in the future. b. The number of firms selling televisions decreases. c. Consumers' income decreases, and televisions are a normal good. d. The number of consumers buying televisions decreases.

A

QN=158 (17283) A price floor a. is a legal minimum on the price at which a good can be sold. b. is a legal maximum on the price at which a good can be sold. c. will generally result in a market shortage. d. will benefit the consumer, but hurt the supplier.

C

QN=159 (17287) A payroll tax is a a. fixed number of dollars that every firm must pay to the government for each worker that the firm hires. b. tax that each firm must pay to the government before the firm can hire workers and operate its business. c. tax on the wages that firms pay their workers. d. tax on all wages above the minimum wage.

D

QN=16 (17114) Communist countries worked on the premise that economic well-being could be best attained by a. a market economy. b. a strong reliance on prices and individuals' self-interests. c. a system of large privately-owned firms. d. the actions of government central planners.

B

QN=160 (17290) If the government removes a binding price floor from a market, then the price received by sellers will a. decrease and the quantity sold in the market will decrease. b. decrease and the quantity sold in the market will increase. c. increase and the quantity sold in the market will decrease. d. increase and the quantity sold in the market will increase.

D

QN=161 (17278) Refer to Figure 6-12. The price paid by buyers after the tax is imposed is a. $3. b. $4. c. $5. d. $7.

B

QN=162 (17274) Rent-control laws dictate a. the exact rent that landlords must charge tenants. b. a maximum rent that landlords may charge tenants. c. a minimum rent that landlords may charge tenants. d. a minimum rent and a maximum rent that landlords may charge tenants.

C

QN=163 (17275) A shortage results when a. a nonbinding price ceiling is imposed on a market. b. a nonbinding price ceiling is removed from a market. c. a binding price ceiling is imposed on a market. d. a binding price ceiling is removed from a market.

B

QN=164 (17279) A government-imposed maximum price at which a good can be sold is called a price a. floor. b. ceiling. c. support. d. equilibrium.

B

QN=165 (17293) Policymakers use taxes a. to raise revenue for public purposes, but not to influence market outcomes. b. both to raise revenue for public purposes and to influence market outcomes. c. when they realize that price controls alone are insufficient to correct market inequities. d. only in those markets in which the burden of the tax falls clearly on the sellers.

D

QN=166 (17272) When a binding price floor is imposed on a market a. (i) price no longer serves as a rationing device. b. (ii) the quantity supplied at the price floor exceeds the quantity that would have been supplied without the price floor. c. (iii) only some sellers benefit. d. All of (i), (ii), and (iii) are correct.

C

QN=167 (17280) Refer to Figure 6-2. If the government imposes a binding price ceiling of $8.00 in this market, the result would be a a. surplus of 20. b. surplus of 40. c. shortage of 20. d. shortage of 40.

C

QN=168 (17294) A surplus results when a. a nonbinding price floor is imposed on a market. b. a nonbinding price floor is removed from a market. c. a binding price floor is imposed on a market. d. a binding price floor is removed from a market.

A

QN=169 (17266) Under rent control, bribery is a mechanism to a. bring the total price of an apartment (including the bribe) closer to the equilibrium price. b. allocate housing to the poorest individuals in the market. c. force the total price of an apartment (including the bribe) to be less than the market price. d. allocate housing to the most deserving tenants.

A

QN=17 (17120) Productivity is defined as the a. amount of goods and services produced from each unit of labor input. b. number of workers required to produce a given amount of goods and services. c. amount of labor that can be saved by replacing workers with machines. d. actual amount of effort workers put into an hour of working time.

B

QN=170 (17295) The imposition of a binding price floor on a market causes quantity demanded to be a. (i) greater than quantity supplied. b. (ii) less than quantity supplied. c. (iii) equal to quantity supplied. d. Both (i) and (ii) are possible.

B

QN=171 (17292) A tax on the sellers of popcorn a. increases the size of the popcorn market. b. decreases the size of the popcorn market. c. has no effect on the size of the popcorn market. d. may increase, decrease, or have no effect on the size of the popcorn market.

A

QN=172 (17273) When a tax is levied on buyers of tea a. buyers of tea and sellers of tea both are made worse off. b. buyers of tea are made worse off and the well-being of sellers is unaffected. c. buyers of tea are made worse off and sellers of tea are made better off. d. the well-being of both buyers of tea and sellers of tea is unaffected.

A

QN=173 (17304) Refer to Figure 6-12. How much tax revenue does this tax produce for the government? a. $24 b. $30 c. $32 d. $56

A

QN=174 (17301) Suppose there is currently a tax of $50 per ticket on airline tickets. Buyers of airline tickets are required to pay the tax to the government. If the tax is reduced from $50 per ticket to $30 per ticket, then a. the demand curve will shift upward by $20, and the price paid by buyers will decrease by less than $20. b. the demand curve will shift upward by $20, and the price paid by buyers will decrease by $20. c. the supply curve will shift downward by $20, and the effective price received by sellers will increase by less than $20. d. the supply curve will shift downward by $20, and the effective price received by sellers will increase by $20.

C

QN=175 (17284) If a price floor is a binding constraint on a market, then a. the equilibrium price must be above the price floor. b. the quantity demanded must exceed the quantity supplied. c. sellers cannot sell all they want to sell at the price floor. d. buyers cannot buy all they want to buy at the price floor.

B

QN=176 (17268) If the minimum wage exceeds the equilibrium wage, then a. the quantity demanded of labor will exceed the quantity supplied. b. the quantity supplied of labor will exceed the quantity demanded. c. the minimum wage will not be binding. d. there will be no unemployment.

C

QN=177 (17300) A $0.10 tax levied on the buyers of socks will cause the a. supply curve for socks to shift down by $0.10. b. supply curve for socks to shift up by $0.10. c. demand curve for socks to shift down by $0.10. d. demand curve for socks to shift up by $0.10.

B

QN=178 (17271) Rent control a. serves as an example of how a social problem can be alleviated or even solved by government policies. b. serves as an example of a price ceiling. c. is regarded by most economists as an efficient way of helping the poor. d. is the most efficient way to allocate scarce housing resources.

A

QN=179 (17286) Suppose sellers of liquor are required to send $1.00 to the government for every bottle of liquor they sell. Further, suppose this tax causes the price paid by buyers of liquor to rise by $0.60 per bottle. Which of the following statements is correct? a. (i) The effective price received by sellers is $0.40 per bottle less than it was before the tax. b. (ii) Sixty percent of the burden of the tax falls on sellers. c. (iii) This tax causes the demand curve for liquor to shift downward by $1.00 at each quantity of liquor. d. All of (i), (ii), and (iii) are correct.

A

QN=18 (17116) To promote good economic outcomes, policymakers should strive to enact policies that a. (i) enhance productivity. b. (ii) enhance individuals' market power. c. (iii) result in a rapidly-growing quantity of money. d. All of (i), (ii), and (iii) are correct.

A

QN=180 (17291) A $2.00 tax levied on the sellers of mailboxes will shift the supply curve a. upward by exactly $2.00. b. upward by less than $2.00. c. downward by exactly $2.00. d. downward by less than $2.00.

C

QN=181 (17302) In the final analysis, tax incidence a. depends on the legislated burden. b. is entirely random. c. depends on the forces of supply and demand. d. falls entirely on buyers or entirely on sellers.

B

QN=182 (17299) When a tax is placed on the sellers of a product, a. buyers pay more and sellers receive more than they did before the tax. b. buyers pay more and sellers receive less than they did before the tax. c. buyers pay less and sellers receive more than they did before the tax. d. buyers pay less and sellers receive less than they did before the tax.

B

QN=183 (17267) Suppose the government has imposed a price floor on cellular phones. Which of the following events could transform the price floor from one that is binding to one that is not binding? a. Cellular phones become less popular. b. Traditional land line phones become more expensive. c. The components used to produce cellular phones become less expensive. d. Firms expect the price of cellular phones to fall in the future.

B

QN=184 (17281) In the housing market, rent controls cause quantity supplied to a. fall and quantity demanded to fall. b. fall and quantity demanded to rise. c. rise and quantity demanded to fall. d. rise and quantity demanded to rise.

A

QN=185 (17269) When a tax is imposed on the sellers of a good, the supply curve shifts a. upward by the amount of the tax. b. downward by the amount of the tax. c. upward by less than the amount of the tax. d. downward by less than the amount of the tax.

A

QN=186 (17298) Suppose sellers of liquor are required to send $1.00 to the government for every bottle of liquor they sell. Further, suppose this tax causes the price paid by buyers of liquor to rise by $0.60 per bottle. Which of the following statements is correct? a. (i) The effective price received by sellers is $0.40 per bottle less than it was before the tax. b. (ii) Sixty percent of the burden of the tax falls on sellers. c. (iii) This tax causes the demand curve for liquor to shift downward by $1.00 at each quantity of liquor. d. All of (i), (ii), and (iii) are correct.

B

QN=187 (17288) Which of the following is correct? a. A tax burden falls more heavily on the side of the market that is more elastic. b. A tax burden falls more heavily on the side of the market that is less elastic. c. A tax burden falls more heavily on the side of the market that is closer to unit elastic. d. A tax burden is distributed independently of the relative elasticities of supply and demand.

D

QN=188 (17277) The goal of rent control is to a. facilitate controlled economic experiments in urban areas. b. help landlords by assuring them a low vacancy rate for their apartments. c. help the poor by assuring them an adequate supply of apartments. d. help the poor by making housing more affordable.

D

QN=189 (17289) A legal maximum on the price at which a good can be sold is called a price a. floor. b. subsidy. c. support. d. ceiling.

B

QN=19 (17139) Which of the following principles is not one of the four principles of individual decision making? a. People face tradeoffs. b. Trade can make everyone better off. c. People respond to incentives. d. Rational people think at the margin.

C

QN=190 (17296) Minimum-wage laws dictate the a. average price employers must pay for labor. b. highest price employers may pay for labor. c. lowest price employers may pay for labor. d. the highest and lowest prices employers may pay for labor.

D

QN=196 (17320) Willingness to pay measures the a. amount a buyer is willing to pay for a good minus the amount the buyer actually pays for it. b. amount a seller actually receives for a good minus the minimum amount the seller is willing to accept. c. maximum amount a buyer is willing to pay minus the minimum amount a seller is willing to accept. d. maximum amount that a buyer will pay for a good.

C

QN=197 (17336) 2. Refer to Figure 7-2. When the price is P1, consumer surplus is a. A. b. A+B. c. A+B+C. d. A+B+D.

B

QN=198 (17329) 5. Refer to Table 7-9. The equilibrium price is a. $10.00. b. $8.00. c. $6.00. d. $4.00.

B

QN=199 (17325) Consumer surplus a. is the amount of a good that a consumer can buy at a price below equilibrium price. b. is the amount a consumer is willing to pay minus the amount the consumer actually pays. c. is the number of consumers who are excluded from a market because of scarcity. d. measures how much a seller values a good.

B

QN=2 (17122) Mitch has $100 to spend and wants to buy either a new amplifier for his guitar or a new mp3 player to listen to music while working out. Both the amplifier and the mp3 player cost $100, so he can only buy one. This illustrates the basic concept that a. trade can make everyone beBtter off. b. people face trade-offs c. rational people think at the margin. d. people respond to incentives.

A

QN=20 (17127) Prices direct economic activity in a market economy by a. influencing the actions of buyers and sellers. b. reducing scarcity of the goods and services produced. c. eliminating the need for government intervention. d. allocating goods and services produced in the most equitable way.

C

QN=200 (17311) Consumer surplus is the a. amount of a good consumers get without paying anything. b. amount a consumer pays minus the amount the consumer is willing to pay. c. amount a consumer is willing to pay minus the amount the consumer actually pays. d. value of a good to a consumer.

A

QN=201 (17343) Refer to Figure 7-5. What is the consumer surplus if the price is $100? a. $2,500 b. $5,000 c. $10,000 d. $20,000

D

QN=202 (17332) Inefficiency can be caused in a market by the presence of a. (i) market power. b. (ii) externalities. c. (iii) imperfectly competitive markets. d. All of (i), (ii), and (iii) are correct.

A

QN=203 (17314) Refer to Table 7-9. Both the demand curve and the supply curve are straight lines. If the price is $8 but only 4 units are bought and sold, producer surplus will be a. $24. b. $28. c. $32. d. $40.

A

QN=204 (17306) Donald produces nails at a cost of $200 per ton. If he sells the nails for $350 per ton, his producer surplus per ton is a. $150. b. $200. c. $350. d. $550.

D

QN=205 (17335) Which of the following will cause a decrease in consumer surplus? a. an increase in the number of sellers of the good b. a decrease in the production cost of the good c. sellers expect the price of the good to be lower next month d. the imposition of a binding price floor in the market

A

QN=206 (17326) Total surplus is a. equal to producer surplus plus consumer surplus. b. equal to the total cost to sellers minus the total value to buyers. c. equal to consumers' willingness to pay plus producers' cost. d. greater than the sum of consumer surplus plus producer surplus.

A

QN=207 (17309) Refer to Figure 7-15. At the equilibrium price, consumer surplus is a. $480. b. $640. c. $1,120. d. $1,280.

B

QN=208 (17330) Which of the Ten Principles of Economics does welfare economics explain more fully? a. The cost of something is what you give up to get it. b. Markets are usually a good way to organize economic activity. c. Trade can make everyone better off. d. A country's standard of living depends on its ability to produce goods and services.

A

QN=209 (17333) Externalities are a. side effects passed on to a party other than the buyers and sellers in the market. b. side effects of government intervention in markets. c. external forces that cause the price of a good to be higher than it otherwise would be. d. external forces that help establish equilibrium price.

D

QN=21 (17121) In considering how to allocate its scarce resources among its various members, a household considers a. (i) each member's abilities. b. (ii) each member's efforts. c. (iii) each member's desires. d. all of (i), (ii), and (iii).

A

QN=210 (17308) Which of the following is correct? a. Efficiency deals with the size of the economic pie, and equality deals with how fairly the pie is sliced. b. Equality can be judged on positive grounds whereas efficiency requires normative judgments. c. Efficiency is more difficult to evaluate than equality. d. Equality and efficiency are both maximized in a society when total surplus is maximized.

C

QN=211 (17322) 3. Refer to Table 7-1. If the table represents the willingness to pay of 4 buyers and the price of the product is $15, then who would be willing to purchase the product? a. Mike b. Mike and Sandy c. Mike, Sandy, and Jonathan d. Mike, Sandy, Jonathan, and Haley

A

QN=212 (17345) Producer surplus measures the a. benefits to sellers of participating in a market. b. costs to sellers of participating in a market. c. price that buyers are willing to pay for sellers' output of a good or service. d. benefit to sellers of producing a greater quantity of a good or service than buyers demand.

A

QN=213 (17340) Consumer surplus is equal to the a. Value to buyers - Amount paid by buyers. b. Amount paid by buyers - Costs of sellers. c. Value to buyers - Costs of sellers. d. Value to buyers - Willingness to pay of buyers.

B

QN=214 (17328) Raisins and milk are complementary goods. An increase in supply of raisins will a. increase consumer surplus in the market for raisins and decrease producer surplus in the market for milk. b. increase consumer surplus in the market for raisins and increase producer surplus in the market for milk. c. decrease consumer surplus in the market for raisins and increase producer surplus in the market for milk. d. decrease consumer surplus in the market for raisins and decrease producer surplus in the market for milk.

D

QN=221 (17305) In which of the following circumstances would a buyer be indifferent about buying a good? a. (i) The amount of consumer surplus the buyer would experience as a result of buying the good is zero. b. (ii) The price of the good is equal to the buyer's willingness to pay for the good. c. (iii) The price of the good is equal to the value the buyer places on the good. d. All of (i), (ii), and (iii) are correct.

B

QN=222 (17318) Suppose Larry, Moe and Curly are bidding in an auction for a mint-condition video of Charlie Chaplin's first movie. Each has in mind a maximum amount that he will bid. This maximum is called a. a resistance price. b. willingness to pay. c. consumer surplus. d. producer surplus.

A

QN=223 (17324) Market efficiency occurs when a. total surplus is maximized. b. producer surplus is maximized. c. all resources are being used. d. consumer surplus equals producer surplus.

D

QN=224 (17319) Which of the following statements is not correct? a. A seller would be eager to sell her product at a price higher than her cost. b. A seller would refuse to sell her product at a price lower than her cost. c. A seller would be indifferent about selling her product at a price equal to her cost. d. Since sellers cannot set the price for their product, they must be willing to sell their product at any price.

A

QN=225 (17321) Belva is willing to pay $65.00 for a pair of shoes for a formal dance. She finds a pair at her favorite outlet shoe store for $48.00. Belva's consumer surplus is a. $17. b. $31. c. $48. d. $65.

A

QN=226 (17313) Brock is willing to pay $400 for a new suit, but he is able to buy the suit for $350. His consumer surplus is a. $50. b. $150. c. $350. d. $400.

B

QN=227 (17307) Which tools allow economists to determine if the allocation of resources determined by free markets is desirable? a. profits and costs to firms b. consumer and producer surplus c. the equilibrium price and quantity d. incomes of and prices paid by buyers

B

QN=228 (17342) On a graph, consumer surplus is represented by the area a. between the demand and supply curves. b. below the demand curve and above the price. c. below the price and above the supply curve. d. below the demand curve and to the right of equilibrium price.

D

QN=229 (17341) In a market, the marginal buyer is the buyer a. whose willingness to pay is higher than that of all other buyers and potential buyers. b. whose willingness to pay is lower than that of all other buyers and potential buyers. c. who is willing to buy exactly one unit of the good. d. who would be the first to leave the market if the price were any higher.

C

QN=23 (17113) As a result of a successful attempt by government to cut the economic pie into more equal slices, a. it is easier to cut the pie, and therefore the economy can produce a larger pie. b. those who earn more income pay less in taxes. c. the pie gets smaller, and there will be less pie overall. d. government will spend too much time cutting and it causes the economy to lose the ability to produce enough pie for everyone.

A

QN=230 (17315) Consumer surplus is a. the amount a buyer is willing to pay for a good minus the amount the buyer actually pays for it. b. the amount a buyer is willing to pay for a good minus the cost of producing the good. c. the amount by which the quantity supplied of a good exceeds the quantity demanded of the good. d. a buyer's willingness to pay for a good plus the price of the good.

B

QN=231 (17344) Refer to Figure 7-5. What happens to the consumer surplus if the price rises from $100 to $150? a. The new consumer surplus is half of the original consumer surplus. b. The new consumer surplus is 25 percent of the original consumer surplus. c. The new consumer surplus is double the original consumer surplus. d. The new consumer surplus is triple the original consumer surplus.

B

QN=232 (17310) Which of the Ten Principles of Economics does welfare economics explain more fully? a. The cost of something is what you give up to get it. b. Markets are usually a good way to organize economic activity. c. Trade can make everyone better off. d. A country's standard of living depends on its ability to produce goods and services.

D

QN=233 (17334) When a buyer's willingness to pay for a good is equal to the price of the good, the a. buyer's consumer surplus for that good is maximized. b. buyer will buy as much of the good as the buyer's budget allows. c. price of the good exceeds the value that the buyer places on the good. d. buyer is indifferent between buying the good and not buying it.

A

QN=234 (17327) 3. Refer to Figure 7-16. If the price were P1, producer surplus would be represented by the area a. F. b. F+G. c. D+H+F. d. D+H+F+G+I.

C

QN=235 (17387) Suppose that electricity producers create a negative externality equal to $6 per unit. Further suppose that the government imposes a $8 per-unit tax on the producers. What is the relationship between the after-tax equilibrium quantity and the socially optimal quantity of electricity to be produced? a. They are equal. b. The after-tax equilibrium quantity is greater than the socially optimal quantity. c. The after-tax equilibrium quantity is less than the socially optimal quantity. d. There is not enough information to answer the question.

D

QN=236 (17348) Which of the following is not correct? a. Markets allocate scarce resources with the forces of supply and demand. b. The equilibrium of supply and demand is typically an efficient allocation of resources. c. Governments can sometimes improve market outcomes. d. Externalities cannot be positive.

C

QN=237 (17352) An externality is the impact of a. society's decisions on the well-being of society. b. a person's actions on that person's well-being. c. one person's actions on the well-being of a bystander. d. society's decisions on the poorest person in the society.

C

QN=238 (17349) Refer to Figure 10-2. Suppose that the production of plastic creates a social cost which is depicted in the graph above. What is the socially optimal quantity of plastic? a. 200 units b. 450 units c. 500 units d. 650 units

B

QN=239 (17382) The difference between social cost and private cost is a measure of the a. loss in profit to the seller as the result of a negative externality. b. cost of an externality. c. cost reduction when the negative externality is eliminated. d. cost incurred by the government when it intervenes in the market.

A

QN=240 (17351) Which of the following statements is correct? a. (i) Corrective taxes are often preferred over direct regulation because they typically reduce externalities at a lower cost. b. (ii) Corrective taxes distort economic incentives. c. (iii) Corrective taxes are often preferred over direct regulation because they typically reduce externalities at a faster rate. d. Both (i) and (ii) are correct.

A

QN=241 (17357) Which of the following is the best statement about markets? a. Markets are usually a good way to organize economic activity. b. Markets are generally inferior to central planning as a way to organize economic activity. c. Markets fail and are therefore not an acceptable way to organize economic activity. d. Markets are a good way to organize economic activity in developed nations, but not in less-developed nations.

B

QN=242 (17354) A paper plant produces water pollution during the production process. If the government forces the plant to internalize the negative externality, then the a. supply curve for paper would shift to the right. b. supply curve for paper would shift to the left. c. demand curve for paper would shift to the right. d. demand curve for paper would shift to the left.

C

QN=243 (17372) When a market is characterized by an externality, the government a. can correct the market failure only in the case of positive externalities. b. can correct the market failure only in the case of negative externalities. c. can correct the market failure in the case of both positive and negative externalities by inducing market participants to internalize the externality. d. cannot correct for externalities due to the existence of patents.

A

QN=244 (17366) If the government were to limit the release of air-pollution produced by a steel mill to 75 parts per million, the policy would be considered a a. regulation. b. corrective tax. c. subsidy. d. market-based policy.

D

QN=245 (17386) Corrective taxes that are imposed upon the producer of a nasty smell can be successful in reducing that smell because the tax makes the producer a. externalize the positive externality. b. externalize the negative externality. c. internalize the positive externality. d. internalize the negative externality.

D

QN=246 (17364) Refer to Figure 10-11. This graph shows the market for pollution when permits are issued to firms and traded in the marketplace. In the absence of a pollution permit system, the quantity of pollution would be a. 25 b. 50 c. 75 d. 100

C

QN=247 (17368) If a sawmill creates too much noise for local residents, a. noise restrictions will force residents to move out of the area. b. a sense of social responsibility will cause owners of the mill to reduce noise levels. c. the government can raise economic well-being through noise-control regulations. d. the government should avoid intervening because the market will allocate resources efficiently.

C

QN=248 (17384) Refer to Figure 10-7. Which quantity represents the social optimum for this market? a. Q1. b. Q2. c. Q3. d. Q4.

A

QN=249 (17367) The term market failure refers to a. a market that fails to allocate resources efficiently. b. an unsuccessful advertising campaign which reduces demand. c. ruthless competition among firms. d. a firm that is forced out of business because of losses.

A

QN=25 (17128) When a society cannot produce all the goods and services people wish to have it is said that the economy is experiencing a. scarcity. b. communism. c. externalities. d. market failure.

C

QN=250 (17355) The supply curve for a product reflects the a. willingness to pay of the marginal buyer. b. quantity buyers will ultimately purchase of the product. c. cost to sellers of producing the product. d. seller's profit from producing the product.

C

QN=251 (17378) Which of the following policies is the government most inclined to use when faced with a positive externality? a. taxation b. permits c. subsidies d. usage fees

B

QN=252 (17374) Positive externalities a. result in a larger than efficient equilibrium quantity. b. result in smaller than efficient equilibrium quantity. c. result in an efficient equilibrium quantity. d. can be internalized with a corrective tax.

B

QN=253 (17389) Mary and Cathy are roommates. Mary assigns a $30 value to smoking cigarettes. Cathy values smoke-free air at $15. Which of the following scenarios is a successful example of the Coase theorem? a. Cathy offers Mary $20 not to smoke. Mary accepts and does not smoke. b. Mary pays Cathy $16 so that Mary can smoke. c. Mary pays Cathy $14 so that Mary can smoke. d. Cathy offers Mary $15 not to smoke. Mary accepts and does not smoke.

B

QN=254 (17360) The height of the demand curve shows a. how much each buyer in the market is willing to pay. b. the willingness to pay of the marginal buyer. c. the maximum price all buyers will pay for a product. d. the lowest price buyers will pay for a product.

B

QN=255 (17369) All remedies for externalities share the goal of a. moving the allocation of resources toward the market equilibrium. b. moving the allocation of resources toward the socially optimal equilibrium. c. increasing the allocation of resources. d. decreasing the allocation of resources.

B

QN=256 (17381) If a paper manufacturer does not bear the entire cost of the dioxin it emits, it will a. emit a lower level of dioxin than is socially efficient. b. emit a higher level of dioxin than is socially efficient. c. emit an acceptable level of dioxin. d. not emit any dioxin in an attempt to avoid paying the entire cost.

B

QN=257 (17356) This figure reflects the market for outdoor concerts in a public park surrounded by residential neighborhoods. Refer to Figure 10-3. What price and quantity combination best represents the optimum price and number of concerts that should be organized? a. P1, Q1 b. P2, Q0 c. P2, Q1 d. The optimum quantity is zero concerts as long as residents in surrounding neighborhoods are adversely affected by noise and congestion.

B

QN=258 (17358) The "invisible hand" leads a market to maximize a. producer profit from that market. b. total benefit to society from that market. c. both equity and efficiency in that market. d. output of goods or services in that market.

B

QN=262 (17370) Suppose that an MBA degree creates no externality because the benefits of an MBA are internalized by the student in the form of higher wages. If the government offers subsidies for MBAs, then which of the following statements is correct? a. The equilibrium quantity of MBAs will equal the socially optimal quantity of MBAs. b. The equilibrium quantity of MBAs will be greater than the socially optimal quantity of MBAs. c. The equilibrium quantity of MBAs will be less than the socially optimal quantity of MBAs. d. There is not enough information to answer the question.

D

QN=263 (17388) The Coase theorem states that a. taxes are an efficient way for governments to remedy negative externalities. b. subsidies are an efficient way for governments to remedy positive externalities. c. industrial policies encourage technology spillovers. d. in the absence of transaction costs, private parties can solve the problem of externalities on their own.

D

QN=264 (17350) In the case of a technology spillover, the government can encourage firms to internalize a positive externality by a. taxing production, which would decrease supply. b. taxing production, which would increase supply. c. subsidizing production, which would decrease supply. d. subsidizing production, which would increase supply.

C

QN=265 (17373) Refer to Figure 10-5. Which of the following statements is correct? a. The marginal benefit of the positive externality is measured by P3 - P1. b. The marginal cost of the negative externality is measured by P3 - P2. c. The marginal cost of the negative externality is measured by P3 - P1. d. The marginal cost of the negative externality is measured by P3 - P0.

C

QN=266 (17377) Suppose that a steel factory emits a certain amount of air pollution, which constitutes a negative externality. If the market does not internalize the externality, a. the supply curve would adequately reflect the marginal social cost of production. b. consumers will be required to pay a higher price for steel than they would have if the externality were internalized. c. the market equilibrium quantity will not be the socially optimal quantity. d. producers will produce less steel than they otherwise would if the externality were internalized.

D

QN=267 (17365) Refer to Figure 10-6. How large would a corrective tax need to be to move this market from the equilibrium outcome to the socially-optimal outcome? a. An amount equal to P' minus P. b. An amount equal to P'. c. An amount equal to P. d. An amount equal to the external cost.

B

QN=268 (17361) When the social cost curve is above a product's supply curve we know that a. government has intervened in the market. b. a negative externality exists in the market. c. a positive externality exists in the market. d. the market reached equilibrium on its own.

B

QN=269 (17347) An externality is a. the costs that parties incur in the process of agreeing and following through on a bargain. b. the uncompensated impact of one person's actions on the well-being of a bystander. c. the proposition that private parties can bargain without cost over the allocation of resources. d. a market equilibrium tax.

D

QN=27 (17124) Most economists believe that an increase in the quantity of money results in a. (i) an increase in the demand for goods and services. b. (ii) lower unemployment in the short run. c. (iii) higher inflation in the long run. d. All of (i), (ii), and (iii) are correct.

A

QN=270 (17379) The best remedy for market failure is often a. a market-based solution. b. shutdown of the market. c. no government intervention. d. externalizing the externalities.

D

QN=271 (17375) Which of the following is NOT a way of internalizing technology spillovers? a. subsidies b. patent protection c. industrial policy d. taxes

C

QN=272 (17363) A positive externality a. is a benefit to the producer of the good. b. is a benefit to the consumer of the good. c. is a benefit to someone other than the producer and consumer of the good. d. results in an optimal level of output.

A

QN=273 (17383) This figure reflects the market for outdoor concerts in a public park surrounded by residential neighborhoods. Refer to Figure 10-3. The social cost curve is above the supply curve because a. it takes into account the external costs imposed on society by the concert. b. it takes into account the effect of local noise restrictions on concerts in parks surrounded by residential neighborhoods. c. concert tickets are likely to cost more than the concert actually costs the organizers. d. residents in the surrounding neighborhoods get to listen to the concert for free.

A

QN=274 (17353) Externalities tend to cause markets to be a. inefficient. b. unequal. c. unnecessary. d. overwhelmed.

A

QN=275 (17371) All externalities a. cause markets to fail to allocate resources efficiently. b. cause equilibrium prices to be too high. c. benefit producers at the expense of consumers. d. cause equilibrium prices to be too low.

D

QN=276 (17380) Which of the following is a way to address an externality problem? a. (i) command and control solution b. (ii) corrective tax c. (iii) corrective subsidy d. all of (i), (ii), and (iii).

B

QN=277 (17385) Refer to Figure 10-7. To internalize the externality in this market, the government should a. impose a tax on this product. b. provide a subsidy for this product. c. forbid production. d. produce the product itself.

A

QN=278 (17408) An ice cream cone is a. excludable and rival in consumption. b. excludable and not rival in consumption. c. not excludable and rival in consumption. d. not excludable and not rival in consumption.

A

QN=279 (17399) Goods that are nonexcludable and nonrival are a. public goods. b. private goods. c. natural monopolies. d. common resources.

A

QN=28 (17125) The terms equality and efficiency are similar in that they both refer to benefits to society. However they are different in that a. equality refers to uniform distribution of those benefits and efficiency refers to maximizing benefits from scarce resources. b. equality refers to maximizing benefits from scarce resources and efficiency refers to uniform distribution of those benefits. c. equality refers to everyone facing identical tradeoffs and efficiency refers to the opportunity cost of the benefits. d. equality refers to the opportunity cost of the benefits and efficiency refers to everyone facing identical tradeoffs.

D

QN=284 (17423) A lighthouse is typically considered to be a public good because a. the owner of the lighthouse is able to exclude beneficiaries from enjoying the lighthouse. b. there is rarely another lighthouse nearby to provide competition. c. a nearby port authority cannot avoid paying fees to the lighthouse owner. d. all passing ships are able to enjoy the benefits of the lighthouse without paying.

A

QN=285 (17424) The Tragedy of the Commons occurs because a. a common resource is rival in consumption. b. a common resource is underutilized. c. crimes are committed in public places. d. common resources are subject to exclusionary rules.

A

QN=286 (17426) Because elephants roam freely in many countries in Africa, each individual African elephant poacher has a. (i) a strong incentive to kill as many elephants as he can find. b. (ii) a strong incentive to protect the elephants. c. (iii) the ability to save the elephants. d. None of (i), (ii), and (iii) is correct.

B

QN=287 (17419) When property rights are not well established, a. private goods become public goods. b. markets fail to allocate resources efficiently. c. the distribution of private goods is unfair. d. government resources are used inefficiently.

C

QN=288 (17410) Which of the following is an example of the free-rider problem? a. Both Fred and Wilma receive low-cost dental care at the local dental school, so neither of them pays the full cost of the care. b. Elmer receives a free lunch from the local "Meals on Wheels" program because of his low monthly income. Yet his next door neighbor, Dorothy, is not eligible for the free lunch. c. Max owns Fido, a large dog who barks whenever anyone walks near his house. Sally lives next to Max, and Fido's barking can be heard whenever anyone walks near her house, too. Thus, Sally receives free protection from burglars because of Fido's barking. d. David purchases a burger at a fast food restaurant and gets a second burger free because the restaurant is having a buy one, get one free sale.

A

QN=289 (17400) Common resources are both a. rival and nonexcludable. b. rival and excludable. c. nonrival and excludable. d. nonrival and nonexcludable.

D

QN=29 (17133) If Japan chooses to engage in trade, it a. will only benefit if it trades with countries that produce goods Japan cannot produce. b. cannot benefit if it trades with less developed countries. c. should first attempt to produce the good itself. d. can benefit by trading with any other country.

B

QN=290 (17413) Which of the following is not a public good? a. national defense b. patented technological knowledge c. general knowledge d. the elimination of poverty

D

QN=291 (17406) Most goods in the economy are a. natural monopolies. b. common resources. c. public goods. d. private goods.

C

QN=292 (17396) A stairwell in a certain office building is always congested at 12:00 p.m. and 2:00 p.m. The congestion is so bad that people have been complaining to the building's owner. Which of the following methods would be the most efficient way of reducing congestion? a. Assign each person in the building a time when they are allowed to use the stairwell. b. Encourage people to voluntarily keep off the stairwell during peak times. c. Charge everyone who uses the stairwell when it is congested the same fee which is high enough to discourage some people from using the stairwell during peak times. People who value the use of the stairs the most will be the ones who use the stairwell at peak times. d. Hold a lottery to determine who wins the right to use the stairwell at peak times.

C

QN=293 (17403) Which of the following quotations illustrates the Tragedy of the Commons? a. "A bird in the hand is worth two in the bush." b. "The only difference between the rich and other people is that the rich have more money." c. "What is common to many is taken least care of, for all men have greater regard for what is their own than for what they possess in common with others." d. "Anyone who is not a socialist before he is 30 has no heart; anyone who is still a socialist after he is 30 has no head."

C

QN=294 (17402) Goods that are not excludable include both a. private goods and public goods. b. natural monopolies and common resources. c. common resources and public goods. d. private goods and natural monopolies.

A

QN=295 (17397) If one person's use of a good diminishes another person's enjoyment of it, the good is a. rival. b. excludable. c. normal. d. exhaustible.

B

QN=296 (17392) When property rights are not well established, a. private goods become public goods. b. markets fail to allocate resources efficiently. c. the distribution of private goods is unfair. d. government resources are used inefficiently.

C

QN=297 (17414) The free-rider problem a. forces the supply of a public good to exceed its demand. b. results in common resources becoming natural monopolies. c. explains why many local governments supply public goods. d. results in public goods becoming private goods.

C

QN=298 (17391) A view of a spectacular sunset along a private beach is an example of a a. private good. b. public good. c. nonrival but excludable good. d. rival but nonexcludable good.

A

QN=299 (17416) Advocates of antipoverty programs believe that fighting poverty a. can make everyone better off. b. is most successfully accomplished by charities. c. is most efficiently accomplished by the market. d. reduces the well-being of tax payers.

B

QN=3 (17123) Senator Smart, who understands economic principles, is trying to convince workers in her district that trade with other countries is beneficial. Senator Smart should argue that trade can be beneficial a. only if it allows us to obtain things that we couldn't make for ourselves. b. because it allows specialization, which increases total output. c. to us if we can gain and the others involved in the trade lose. d. in only a limited number of circumstances because others are typically self-interested.

A

QN=30 (17118) When society requires that firms reduce pollution, there is a. a tradeoff because of reduced incomes to the firms' owners and workers. b. a tradeoff only if some firms are forced to close. c. no tradeoff, since the cost of reducing pollution falls only on the firms affected by the requirements. d. no tradeoff, since everyone benefits from reduced pollution.

C

QN=304 (17418) Suppose that you want to put on a fireworks display in your hometown of 1,000 people this July. The cost of the display is $6,000, and each person values the display at $5. After a month, you have only sold 50 tickets at $5 each. The result is that a. the local government should put on the display, but you should not. b. you should still put on the display, but the local government should not. c. neither you nor the local government should put on the display. d. either you or the local government should put on the display.

B

QN=305 (17393) When a good is excludable, a. one person's use of the good diminishes another person's ability to use it. b. people can be prevented from using the good. c. no more than one person can use the good at the same time. d. everyone will be excluded from using the good.

A

QN=306 (17401) Suppose a human life is worth $10 million. Installing a better lighting system in the city park would reduce the risk of someone being murdered there from 2.6 to 1.9 percent over the life of the system. The city should install the new lighting system if its cost does not exceed a. $70,000. b. $260,000. c. $190,000. d. $10,000,000.

C

QN=307 (17415) Government policy can potentially raise economic well-being a. in all markets for goods and services. b. in economic models, but not in reality. c. when a good does not have a price attached to it. d. never.

B

QN=308 (17420) An FM radio signal is an example of a good that is a. private. b. nonrival in consumption. c. social. d. nonexcludable in production.

A

QN=309 (17421) The value and cost of goods are easiest to determine when the goods are a. private goods. b. public goods. c. common resources. d. natural monopolies.

D

QN=31 (17135) The business cycle is measured by the a. (i) production of goods and services. b. (ii) number of people employed. c. (iii) the interest rate. d. both (i) and (ii).

D

QN=310 (17404) When the absence of property rights causes a market failure, the government can potentially solve the problem a. (i) by clearly defining property rights. b. (ii) through regulation. c. (iii) by supplying the good itself. d. All of (i), (ii), and (iii) are correct.

D

QN=311 (17398) Which of the following would be considered a private good? a. national defense b. a public beach c. local cable television service d. a bottle of natural mineral water

D

QN=312 (17417) If the government decides to build a new highway, the first step would be to conduct a study to determine the value of the project. The study is called a a. fiscal analysis. b. monetary analysis. c. welfare analysis. d. cost-benefit analysis.

B

QN=313 (17395) One way to eliminate the Tragedy of the Commons is to a. increase law enforcement in public areas. b. limit access to the commons. c. increase access to the commons. d. decrease taxes.

C

QN=314 (17411) People have little incentive to produce a public good because a. the social benefit is less than the private benefit. b. the social benefit is less than the social cost. c. there is a free-rider problem. d. there is a Tragedy of the Commons.

C

QN=315 (17431) Table 13-13 Consider the following table of long-run total cost for four different firms: Refer to Table 13-13. Which firm has diseconomies of scale over the entire range of output? a. Firm 1 b. Firm 2 c. Firm 3 d. Firm 4

C

QN=316 (17433) Refer to Table 13-2. The marginal product of the second worker is a. 90 units. b. 85 units. c. 80 units. d. 20 units.

C

QN=317 (17458) A difference between explicit and implicit costs is that a. explicit costs are greater than implicit costs. b. explicit costs do not require a direct monetary outlay by the firm, whereas implicit costs do. c. implicit costs do not require a direct monetary outlay by the firm, whereas explicit costs do. d. implicit costs are greater than explicit costs.

A

QN=318 (17461) Which of the following expressions is correct? a. accounting profit = economic profit + implicit costs b. accounting profit = total revenue - implicit costs c. economic profit = accounting profit + explicit costs d. economic profit = total revenue - implicit costs

B

QN=319 (17444) Which of the following costs would be regarded as an implicit cost? a. the cost of accounting services b. the opportunity cost of financial capital that has been invested in the business c. the cost of compliance with government regulation d. all costs that involve outlays of money by the firm

D

QN=32 (17140) The phenomenon of scarcity stems from the fact that a. most economies' production methods are not very good. b. in most economies, wealthy people consume disproportionate quantities of goods and services. c. governments restrict production of too many goods and services. d. resources are limited.

A

QN=320 (17448) Refer to Figure 13-9. The firm experiences economies of scale at which output levels? a. (i) output levels less than M b. (ii) output levels between M and N c. (iii) output levels greater than N d. All of (i), (ii), and (iii) are correct as long as the firm is operating in the long run.

B

QN=321 (17459) At Bert's Bootery, the total cost of producing twenty pairs of boots is $400. The marginal cost of producing the twenty-first pair of boots is $83. We can conclude that the a. average variable cost of 21 pairs of boots is $23. b. average total cost of 21 pairs of boots is $23. c. average total cost of 21 pairs of boots is $15.09. d. marginal cost of the 20th pair of boots is $20.

C

QN=322 (17439) Which of the following measures of cost is best described as "the cost of a typical unit of output if total cost is divided evenly over all the units produced?" a. average fixed cost b. average variable cost c. average total cost d. marginal cost

D

QN=323 (17467) Refer to Table 13-6. What is the marginal cost of producing the fifth unit of output? a. $4 b. $40 c. $50 d. $70

C

QN=324 (17432) Jane decides to open her own business and earns $50,000 in accounting profit the first year. When deciding to open her own business, she turned down three separate job offers with annual salaries of $30,000, $40,000, and $45,000. What is Jane's economic profit from running her own business? a. $-55,000 b. $-5,000 c. $5,000 d. $20,000

C

QN=331 (17438) John has decided to start his own lawn-mowing business. To purchase the mowers and the trailer to transport the mowers, John withdrew $1,000 from his savings account, which was earning 3% interest, and borrowed an additional $2,000 from the bank at an interest rate of 7%. What is John's annual opportunity cost of the financial capital that has been invested in the business? a. $30 b. $140 c. $170 d. $300

B

QN=332 (17460) Economic profit is equal to a. total revenue minus the explicit cost of producing goods and services. b. total revenue minus the opportunity cost of producing goods and services. c. total revenue minus the accounting cost of producing goods and services. d. average revenue minus the average cost of producing the last unit of a good or service.

C

QN=333 (17454) Profit is defined as total revenue a. plus total cost. b. times total cost. c. minus total cost. d. divided by total cost.

A

QN=334 (17450) Kirsten sells 300 glasses of lemonade at $0.50 each. Her total costs are $125. Her profits are a. $25. b. $124.50. c. $125. d. $150.

C

QN=335 (17456) An example of an opportunity cost that is also an implicit cost is a. (i) a lease payment. b. (ii) the cost of raw materials. c. (iii) the value of the business owner's time. d. All of (i), (ii), and (iii) are correct.

A

QN=336 (17443) When, for a firm, long-run average total cost decreases as the quantity of output increases, we have a situation of a. economies of scale. b. diseconomies of scale. c. coordination problems arising from the large size of the firm. d. fixed costs greatly exceeding variable costs.

D

QN=337 (17445) Refer to Table 13-3. What is total output when 5 workers are hired? a. 70 b. 120 c. 160 d. 190

D

QN=338 (17428) Which of the following measures of cost is best described as "the increase in total cost that arises from an extra unit of production?" a. variable cost b. average variable cost c. average total cost d. marginal cost

A

QN=339 (17427) Larry's Lunchcart is a small street vendor business. If Larry makes 15 pretzels in his first hour of business and incurs a total cost of $16.50, his average total cost per pretzel is a. $1.10. b. $6.50. c. $15.00. d. $16.50.

C

QN=34 (17141) Hue decides to spend three hours working overtime rather than watching a video with her friends. She earns $8 an hour. Her opportunity cost of working is a. the $24 she earns working. b. the $24 minus the enjoyment she would have received from watching the video. c. the enjoyment she would have received had she watched the video. d. nothing, since she would have received less than $24 of enjoyment from the video.

B

QN=340 (17465) Charles's Car Wash has average variable costs of $2 and average total costs of $3 when it produces 100 units of output (car washes). The firm's total variable cost is a. $100. b. $200. c. $300. d. $500.

B

QN=341 (17434) Suppose that for a particular firm the only variable input into the production process is labor and that output equals zero when no workers are hired. In addition, suppose that when the firm hires 2 workers, the total cost of production is $100. When the firm hires 3 workers, the total cost of production is $120. In addition, assume that the variable cost per unit of labor is the same regardless of the number of units of labor that are hired. What is the firm's fixed cost? a. $40 b. $60 c. $80 d. $100

D

QN=342 (17446) Which of the following measures of cost is best described as "the increase in total cost that arises from an extra unit of production?" a. variable cost b. average variable cost c. average total cost d. marginal cost

A

QN=343 (17441) Scenario 13-4 For the following questions, assume that a given firm experiences decreasing marginal product of labor with the addition of each worker regardless of the current output level. Refer to Scenario 13-4. Average variable cost will be a. always rising. b. always falling. c. U-shaped. d. constant.

D

QN=344 (17463) Which of the following statements about a production function is correct for a firm that uses labor to produce output? a. (i) The production function depicts the relationship between the quantity of labor and the quantity of output. b. (ii) The slope of the production function measures marginal product. c. (iii) The slopes of the production function and the total cost curve are inversely related; if one is increasing, the other is decreasing. d. All of (i), (ii), and (iii) are correct.

A

QN=345 (17430) The firm's efficient scale is the quantity of output that minimizes a. average total cost. b. average fixed cost. c. average variable cost. d. marginal cost.

C

QN=346 (17435) Economists normally assume that the goal of a firm is to (i) sell as much of their product as possible. (ii) set the price of the product as high as possible. (iii) maximize profit. a. (i) and (ii) are true. b. (ii) and (iii) are true. c. only (iii) is true. d. (i) and (iii) are true.

B

QN=347 (17468) Refer to Table 13-6. What is the shape of the marginal cost curve for this firm? a. constant b. upward-sloping c. downward-sloping d. U-shaped

A

QN=348 (17437) XYZ corporation produced 300 units of output but sold only 275 of the units it produced and discarded the remaining 25 defected units. The average cost of production for each unit of output produced was $100. Each of the 275 units sold was sold for a price of $95. Total profit for the XYZ corporation would be a. -$3,875. b. $26,125. c. $28,500. d. $30,000.

A

QN=349 (17462) Refer to Figure 13-2. As the number of workers increases, a. (i) marginal product decreases. b. (ii) total output decreases. c. (iii) marginal product increases but at a decreasing rate. d. Both (i) and (ii) are correct.

C

QN=35 (17132) You are considering staying in college another semester so that you can complete a major in economics. In deciding whether or not to stay you should a. compare the total cost of your education to the total benefits of your education. b. compare the total cost of your education to the benefits of staying one more semester. c. compare the cost of staying one more semester to the benefits of staying one more semester. d. compare the total benefits of your education to the cost of staying one more semester.

B

QN=364 (17516) Which of the following statements best reflects the production decision of a profit-maximizing firm in a competitive market when price falls below the minimum of average variable cost? a. The firm will continue to produce to attempt to pay fixed costs. b. The firm will immediately stop production to minimize its losses. c. The firm will stop production as soon as it is able to pay its sunk costs. d. The firm will continue to produce in the short run but will likely exit the market in the long run.

D

QN=365 (17472) 3. Refer to Table 14-7. If the firm is currently producing 14 units, what would you advise the owners? a. decrease quantity to 13 units b. increase quantity to 17 units c. continue to operate at 14 units d. increase quantity to 16 units

B

QN=366 (17489) Which of the following statements regarding a competitive market is not correct? a. There are many buyers and many sellers in the market. b. Because of firm location or product differences, some firms can charge a higher price than other firms and still maintain their sales volume. c. Price and average revenue are equal. d. Price and marginal revenue are equal.

B

QN=367 (17508) In a perfectly competitive market, the market supply curve is a. the marginal cost curve above average total cost for a representative firm. b. the horizontal sum of all the individual firms' supply curves. c. the vertical sum of all the individual firms' supply curves. d. always a horizontal line.

B

QN=368 (17474) In a perfectly competitive market, the market supply curve is a. the marginal cost curve above average total cost for a representative firm. b. the horizontal sum of all the individual firms' supply curves. c. the vertical sum of all the individual firms' supply curves. d. always a horizontal line.

C

QN=369 (17504) Which of the following is not a characteristic of a competitive market? a. Buyers and sellers are price takers. b. Each firm sells a virtually identical product. c. Free entry is limited. d. Each firm chooses an output level that maximizes profits.

D

QN=37 (17162) An economic theory about international trade that is based on the assumption that there are only two countries trading two goods a. is useless, since the real world has many countries trading many goods. b. can be useful only in situations involving two countries and two goods. c. can be useful in the classroom, but is useless in the real world. d. can be useful in helping economists understand the complex world of international trade involving many countries and many goods.

B

QN=370 (17487) A firm in a competitive market has the following cost structure: If the market price is $4, this firm will a. produce two units in the short run and exit in the long run. b. produce three units in the short run and exit in the long run. c. produce four units in the short run and exit in the long run. d. shut down in the short run and exit in the long run.

C

QN=371 (17478) 4. Refer to Figure 14-2. This is a competitive market. If the market price is $10, what is the firm's total cost? a. $15 b. $30 c. $35 d. $50

D

QN=372 (17506) Refer to Table 14-7. If the firm is currently producing 14 units, what would you advise the owners? a. decrease quantity to 13 units b. increase quantity to 17 units c. continue to operate at 14 units d. increase quantity to 16 units

D

QN=373 (17498) The production decisions of perfectly competitive firms follow one of the Ten Principles of Economics, which states that rational people a. consider sunk costs. b. equate prices to the average costs of production. c. will eventually leave markets that experience zero profit. d. think at the margin.

C

QN=374 (17482) Scenario 14-1 Assume a certain firm is producing Q = 1,000 units of output. At Q = 1,000, the firm's marginal cost equals $15 and its average total cost equals $11. The firm sells its output for $12 per unit. Refer to Scenario 14-1. At Q = 999, the firm's profits equal a. $993. b. $997. c. $1,003. d. $1,007.

B

QN=375 (17514) Refer to Figure 14-1. Which of the four prices corresponds to a perfectly competitive firm earning zero economic profits in the short run? a. P1 b. P2 c. P3 d. P4

C

QN=376 (17490) For a certain firm, the 100th unit of output that the firm produces has a marginal revenue of $10 and a marginal cost of $11. It follows that the a. production of the 100th unit of output increases the firm's profit by $1. b. production of the 100th unit of output increases the firm's average total cost by $1. c. firm's profit-maximizing level of output is less than 100 units. d. production of the 110th unit of output must increase the firm's profit by less than $1.

D

QN=377 (17502) Which of the following could be used to calculate the profit for a firm? a. Profit = MR - MC b. Profit = MR - TC c. Profit = (P - MC)*Q d. Profit = (P - ATC)*Q

C

QN=378 (17494) For any competitive market, the supply curve is closely related to the a. preferences of consumers who purchase products in that market. b. income tax rates of consumers in that market. c. firms' costs of production in that market. d. interest rates on government bonds.

B

QN=379 (17475) Which of the following is not a characteristic of a perfectly competitive market? a. Firms are price takers. b. Firms have difficulty entering the market. c. There are many sellers in the market. d. Goods offered for sale are largely the same.

D

QN=38 (17184) Which of the following transactions does not take place in the markets for factors of production in the circular-flow diagram? a. a landowner leases land to a farmer b. a farmer hires a teenager to help with harvest c. a retired farmer sells his combine to a new farmer d. a woman buys corn for dinner

B

QN=380 (17503) 5. Refer to Figure 14-2. The firm will earn zero economic profit if the market price is a. $0 b. $6 c. $7 d. $10

C

QN=381 (17473) Which of the following represents the firm's long-run condition for exiting a market? a. exit if P < MC b. exit if P < FC c. exit if P < ATC d. exit if MR < MC

C

QN=382 (17511) Suppose that in a competitive market the equilibrium price is $2.50. What is marginal revenue for the last unit sold by the typical firm in this market? a. less than $2.50 b. more than $2.50 c. exactly $2.50 d. The marginal revenue cannot be determined without knowing the actual quantity sold by the typical firm.

D

QN=386 (17491) In the figure below, panel (a) depicts the linear marginal cost of a firm in a competitive market, and panel (b) depicts the linear market supply curve for a market with a fixed number of identical firms. Refer to Figure 14-8. If at a market price of $1.75, 52,500 units of output are supplied to this market, how many identical firms are participating in this market? a. 75 b. 100 c. 250 d. 300

C

QN=387 (17470) Which of the following is not a characteristic of a competitive market? a. Buyers and sellers are price takers. b. Each firm sells a virtually identical product. c. Free entry is limited. d. Each firm chooses an output level that maximizes profits.

C

QN=388 (17471) Suppose that a firm operating in a perfectly competitive market sells 400 units of output at a price of $4 each. Which of the following statements is correct? (i) Marginal revenue equals $4. (ii) Average revenue equals $100. (iii) Total revenue equals $1,600. a. (i) only b. (iii) only c. (i) and (iii) only d. (i), (ii), and (iii)

C

QN=389 (17507) Which of the following represents the firm's long-run condition for exiting a market? a. exit if P < MC b. exit if P < FC c. exit if P < ATC d. exit if MR < MC

C

QN=39 (17156) When two variables have a positive correlation, a. (i) when the x-variable increases, the y-variable decreases. b. (ii) when the x-variable decreases, the y-variable increases. c. (iii) when the x-variable increases, the y-variable increases. d. More than one of (i), (ii), and (iii) is correct.

B

QN=390 (17513) Refer to Figure 14-1. If the market price is P3, in the short run, the perfectly competitive firm will earn a. positive economic profits. b. negative economic profits but will try to remain open. c. negative economic profits and will shut down. d. zero economic profits.

D

QN=391 (17510) Which of the following statements is correct? a. For all firms, marginal revenue equals the price of the good. b. Only for competitive firms does average revenue equal the price of the good. c. Marginal revenue can be calculated as total revenue divided by the quantity sold. d. Only for competitive firms does average revenue equal marginal revenue.

A

QN=392 (17477) Refer to Table 14-4. The firm will produce a quantity greater than 4 because at 4 units of output, marginal cost a. is less than marginal revenue. b. equals marginal revenue. c. is greater than marginal revenue. d. is minimized.

A

QN=393 (17480) In a perfectly competitive market, a. no one seller can influence the price of the product. b. price exceeds marginal revenue for each unit sold. c. average revenue exceeds marginal revenue for each unit sold. d. administrative barriers can make it difficult for firms to enter an industry.

C

QN=394 (17512) If a competitive firm is currently producing a level of output at which marginal cost exceeds marginal revenue, then a. (i) average revenue exceeds marginal cost. b. (ii) the firm is earning a positive profit. c. (iii) decreasing output would increase the firm's profit. d. All of (i), (ii), and (iii) are correct.

A

QN=395 (17497) Profit-maximizing firms in a competitive market produce an output level where a. marginal cost equals marginal revenue. b. marginal cost equals average total cost. c. marginal revenue is increasing. d. price is less than marginal revenue.

C

QN=396 (17486) When a restaurant stays open for lunch service even though few customers patronize the restaurant for lunch, which of the following principles is (are) best demonstrated? (i) Fixed costs are sunk in the short run. (ii) In the short run, only fixed costs are important to the decision to stay open for lunch. (iii) If revenue exceeds variable cost, the restaurant owner is making a profitable strategic decision to remain open for lunch. a. (i) and (ii) only b. (ii) and (iii) only c. (i) and (iii) only d. All of these principles are demonstrated.

D

QN=397 (17485) Use the information for a competitive firm in the table below to answer the following questions. Refer to Table 14-5. If the firm finds that its marginal cost is $11, it should a. (i) increase production to maximize profit. b. (ii) increase the price of the product to maximize profit. c. (iii) advertise to attract additional buyers to maximize profit. d. None of (i), (ii), and (iii) are correct.

D

QN=398 (17483) Profit maximizing firms in competitive industries with free entry and exit face a price equal to the lowest possible a. marginal cost of production. b. fixed cost of production. c. total cost of production. d. Average total cost of production.

D

QN=399 (17496) A competitive firm has been selling its output for $20 per unit and has been maximizing its profit, which is positive. Then, the price rises to $25, and the firm makes whatever adjustments are necessary to maximize its profit at the now-higher price. Once the firm has adjusted, which of the following statements is correct? a. (i) The firm's quantity of output is higher than it was previously. b. (ii) The firm's average total cost is higher than it was previously. c. (iii) The firm's marginal revenue is higher than it was previously. d. All of (i), (ii), and (iii) are correct.

A

QN=4 (17136) Economics deals primarily with the concept of a. scarcity. b. money. c. poverty. d. banking.

B

QN=40 (17164) The slope of a steep upward-sloping line will be a a. small positive number. b. large positive number. c. small negative number. d. large negative number.

B

QN=400 (17518) A competitive market is in long-run equilibrium. If demand increases, we can be certain that price will a. rise in the short run. Some firms will enter the industry. Price will then rise to reach the new long-run equilibrium. b. rise in the short run. Some firms will enter the industry. Price will then fall to reach the new long-run equilibrium. c. fall in the short run. All, some, or no firms will shut down, and some of them will exit the industry. Price will then rise to reach the new long-run equilibrium. d. not rise in the short run because firms will enter to maintain the price.

B

QN=401 (17481) Scenario 14-1 Assume a certain firm is producing Q = 1,000 units of output. At Q = 1,000, the firm's marginal cost equals $15 and its average total cost equals $11. The firm sells its output for $12 per unit. Refer to Scenario 14-1. At Q = 1,000, the firm's profits equal a. $-200. b. $1,000 c. $3,000. d. $4,000.

D

QN=418 (17534) Which of the following is an example of a barrier to entry? (i) A key resource is owned by a single firm. (ii) The costs of production make a single producer more efficient than a large number of producers. (iii) The government has given the existing monopoly the exclusive right to produce the good. a. (i) and (ii) b. (ii) and (iii) c. (i) only d. All of these examples are barriers to entry.

B

QN=419 (17536) Refer to Figure 15-1. The shape of the average total cost curve reveals information about the nature of the barrier to entry that might exist in a monopoly market. Which of the following monopoly types best coincides with the figure? a. (i) ownership of a key resource by a single firm b. (ii) natural monopoly c. (iii) government-created monopoly d. None of (i), (ii), and (iii) is correct.

A

QN=42 (17170) The slope of a fairly flat upward-sloping line will be a a. small positive number. b. large positive number. c. small negative number. d. large negative number.

C

QN=420 (17531) Monopolies use their market power to a. charge prices that equal minimum average total cost. b. increase the quantity sold as they increase price. c. charge a price that is higher than marginal cost. d. dump excess supplies of their product on the market.

B

QN=421 (17526) When a monopolist decreases the price of its good, consumers a. continue to buy the same amount. b. buy more. c. buy less. d. may buy more or less, depending on the price elasticity of demand.

D

QN=422 (17547) A monopoly chooses to supply the market with a quantity of a product that is determined by the intersection of the a. marginal cost and demand curves. b. average total cost and demand curves. c. marginal revenue and average total cost curves. d. marginal revenue and marginal cost curves.

A

QN=423 (17549) For a profit-maximizing monopolist, a. P > MR = MC. b. P = MR = MC. c. P > MR > MC. d. MR < MC < P.

C

QN=424 (17545) When a firm has a natural monopoly, the firm's a. marginal cost always exceeds its average total cost. b. total cost curve is horizontal. c. average total cost curve is downward sloping. d. marginal cost curve must lie above the firm's average total cost curve.

A

QN=425 (17560) A perfectly price-discriminating monopolist is able to a. maximize profit and produce a socially-optimal level of output. b. maximize profit, but not produce a socially-optimal level of output. c. produce a socially-optimal level of output, but not maximize profit. d. exercise illegal preferences regarding the race and/or gender of its employees.

B

QN=426 (17524) A perfectly competitive market a. may not be in the best interests of society, whereas a monopoly market promotes general economic well-being b. promotes general economic well-being, whereas a monopoly market may not be in the best interests of society. c. and a monopoly market are equally likely to promote general economic well-being. d. is less likely to promote general economic well-being than a monopoly market.

C

QN=427 (17522) Financial aid to college students, quantity discounts, and senior citizen discounts are all examples of a. consumer surplus. b. deadweight loss. c. price discrimination. d. nonprofit pricing strategies.

C

QN=428 (17559) Refer to Figure 15-8. The deadweight loss caused by a profit-maximizing monopoly amounts to a. $150. b. $200. c. $250. d. $500.

A

QN=429 (17533) Refer to Figure 15-11. If the monopoly firm perfectly price discriminates, then consumer surplus amounts to a. $0. b. $250. c. $500. d. $1,000.

C

QN=43 (17175) Refer to Figure 2-8, Panel (a). In order to gain 2 donuts by moving from point L to point M, society must sacrifice a. (i) efficiency. b. (ii) employment. c. (iii) 4 cups of coffee. d. More than one of (i), (ii), and (iii) is correct.

C

QN=430 (17527) Refer to Figure 15-5. A profit-maximizing monopoly's profit is equal to a. P4 * Q3. b. (P4-P2) * Q3. c. (P4-P1) * Q3. d. (P5-P0) * Q1.

D

QN=431 (17525) Which of the following is not a reason for the existence of a monopoly? a. sole ownership of a key resource b. patents c. copyrights d. diseconomies of scale

D

QN=432 (17537) Refer to Figure 15-2. Profit will be maximized by charging a price equal to a. P0. b. P1. c. P2. d. P3.

A

QN=433 (17532) If a monopolist has zero marginal costs, it will produce a. the output at which total revenue is maximized. b. in the range in which marginal revenue is still increasing. c. at the point at which marginal revenue is at a maximum. d. in the range in which marginal revenue is negative.

D

QN=434 (17541) A monopoly chooses to supply the market with a quantity of a product that is determined by the intersection of the a. marginal cost and demand curves. b. average total cost and demand curves. c. marginal revenue and average total cost curves. d. marginal revenue and marginal cost curves.

C

QN=435 (17520) Which of the following statements is correct? a. The demand curve facing a competitive firm is horizontal, as is the demand curve facing a monopolist. b. The demand curve facing a competitive firm is downward sloping, whereas the demand curve facing a monopolist is horizontal. c. The demand curve facing a competitive firm is horizontal, whereas the demand curve facing a monopolist is downward sloping. d. The demand curve facing a competitive firm is downward sloping, as is the demand curve facing a monopolist.

D

QN=436 (17530) Most markets are not monopolies in the real world because a. firms usually face downward-sloping demand curves. b. supply curves slope upward. c. price is usually set equal to marginal cost by firms. d. there are reasonable substitutes for most goods.

A

QN=454 (17599) Which of the following is a commonly-cited benefit of advertising? a. Advertising can be a signal of the quality of a product. b. Advertising impedes competition. c. Advertising reduces the deadweight loss associated with monopolistic competition. d. Advertising encourages free entry, which increases profits.

C

QN=57 (17174) The production possibilities frontier is a graph that shows the various combinations of output that an economy a. should produce. b. wants to produce. c. can produce. d. demands.

D

QN=437 (17528) Suppose when a monopolist produces 75 units its average revenue is $10 per unit, its marginal revenue is $5 per unit, its marginal cost is $6 per unit, and its average total cost is $5 per unit. What can we conclude about this monopolist? a. The monopolist is currently maximizing profits, and its total profits are $375. b. The monopolist is currently maximizing profits, and its total profits are $300. c. The monopolist is not currently maximizing profits; it should produce more units and charge a lower price to maximize profits. d. The monopolist is not currently maximizing profits; it should produce fewer units and charge a higher price to maximize profits.

D

QN=438 (17543) A movie theater can increase its profits through price discrimination by charging a higher price to adults and a lower price to children if it a. (i) can prevent children from buying the lower-priced tickets and selling them to adults. b. (ii) has some degree of monopoly pricing power. c. (iii) can easily distinguish between the two groups of customers. d. All of (i), (ii), and (iii) are correct.

C

QN=439 (17529) A monopoly market is characterized by a. many buyers and sellers b. "natural" products. c. barriers to entry. d. a Nash equilibrium.

B

QN=44 (17149) Economists sometimes give conflicting advice because a. graduate students in economics are encouraged to argue with each other. b. economists have different values and scientific judgment. c. economists acting as scientists do not like to agree with economists acting as policy advisers. d. economics is more of a belief system than a science.

C

QN=440 (17558) A monopoly market a. always maximizes total economic well-being. b. always minimizes consumer surplus. c. generally fails to maximize total economic well-being. d. generally fails to maximize producer surplus.

B

QN=441 (17523) During the holiday season, high-end retailers frequently place a high price on merchandise on weekends and discount the price during the week. They do this because they believe that two groups of customers exist: shoppers with little free time and bargain hunters. Bargain hunters have time to shop around and frequently shop during the week. What do economists call this price strategy used by high-end retailers? a. oligopoly b. price discrimination c. compensating differential d. in-kind transfers

B

QN=442 (17535) Patent and copyright laws are major sources of a. (i) natural monopolies. b. (ii) government-created monopolies. c. (iii) resource monopolies. d. None of (i), (ii), and (iii) is correct.

A

QN=443 (17519) One difference between a perfectly competitive firm and a monopoly is that a perfectly competitive firm produces where a. marginal cost equals price, while a monopolist produces where price exceeds marginal cost. b. marginal cost equals price, while a monopolist produces where marginal cost exceeds price. c. price exceeds marginal cost, while a monopolist produces where marginal cost equals price. d. marginal cost exceeds price, while a monopolist produces where marginal cost equals price.

B

QN=444 (17554) Suppose most people regard emeralds, rubies, and sapphires as close substitutes for diamonds. Then DeBeers, a large diamond company, has a. less incentive to advertise than it would otherwise have. b. less market power than it would otherwise have. c. more control over the price of diamonds than it would otherwise have. d. higher profits than it would otherwise have.

A

QN=445 (17521) The deadweight loss that arises from a monopoly is a consequence of the fact that the monopoly a. quantity is lower than the socially-optimal quantity. b. price equals marginal revenue. c. price is the same as average revenue. d. earns positive profits.

D

QN=446 (17548) Antitrust laws allow the government to a. (i) prevent mergers. b. (ii) break up companies. c. (iii) promote competition. d. All of (i), (ii), and (iii) are correct.

C

QN=447 (17550) A monopolist will choose to increase output when a. market price increases. b. at all levels of output, marginal cost increases. c. at the present level of output, marginal revenue exceeds marginal cost. d. the demand curve shifts to the left.

D

QN=448 (17546) A profit-maximizing monopolist charges a price of $14. The intersection of the marginal revenue curve and the marginal cost curve occurs where output is 15 units and marginal cost is $7. What is the monopolist's profit? a. $90 b. $105 c. $180 d. Not enough information is given to determine the answer.

A

QN=449 (17582) A monopolistically competitive market is like both a competitive market and a monopoly in that firms in all three market structures a. (i) can earn economic profits in the short run. b. (ii) can earn economic profits in the long run. c. (iii) charge a price above marginal cost. d. All of (i), (ii), and (iii) are correct.

C

QN=45 (17153) The scientific method is a. the use of modern technology to understand the way the world works. b. the use of controlled laboratory experiments to understand the way the world works. c. the dispassionate development and testing of theories about how the world works. d. the search for evidence to support preconceived theories about how the world works.

D

QN=450 (17568) Markets with only a few sellers, each offering a product similar or identical to the others, are typically referred to as a. competitive markets. b. monopoly markets. c. monopolistically competitive markets. d. oligopoly markets.

A

QN=451 (17567) If a firm in a monopolistically competitive market successfully uses advertising to decrease the elasticity of demand for its product, the firm will a. be able to increase its markup over marginal cost. b. eventually have to lower price to remain competitive. c. increase the welfare of society. d. reduce its average total cost.

B

QN=452 (17569) Typical firms in our economy are classified as a. perfectly competitive. b. imperfectly competitive. c. duopolists. d. oligopolists.

C

QN=453 (17584) Select the type of market that is described by the following attributes: many firms, differentiated products, and free entry. a. natural monopoly b. perfectly competition c. monopolistic competition d. monopoly

B

QN=570 (17693) Refer to Figure 21-3. In graph (a), if income is equal to $120, the price of good Y is a. $1. b. $2. c. $4. d. $6.

A

QN=457 (17592) A monopolistically competitive market is like a competitive market in that a. both market structures feature easy entry by new firms in the long run. b. the main objective of firms in both market structures is something other than profit maximization. c. firms in both market structures produce the welfare-maximizing level of output. d. firms in both market structures set price above marginal cost.

B

QN=458 (17562) One way in which monopolistic competition differs from oligopoly is that a. there are no barriers to entry in oligopolies. b. in oligopoly markets there are only a few sellers. c. all firms in an oligopoly eventually earn zero economic profits. d. strategic interactions between firms are rare in oligopolies.

B

QN=459 (17561) Monopolistic competition is a type of a. oligopoly. b. market structure. c. price discrimination. d. advertising strategy.

D

QN=46 (17152) Refer to Figure 2-2. Malika works as an attorney for a corporation and is paid a salary in exchange for the legal services she performs. Jarel owns office buildings and rents his buildings to companies in exchange for rent payments. If Malika's income is represented by a flow of dollars from Box D to Box B of this circular-flow diagram, then Jarel's income is represented by a flow of dollars a. from Box A to Box C. b. from Box C to Box A. c. from Box B to Box D. d. from Box D to Box B.

A

QN=460 (17572) Binding agreements concerning production levels between oligopolists can lead the involved firms to a. monopoly profit. b. lower prices and more profit. c. bankruptcy. d. higher prices and less profit.

B

QN=461 (17565) Which of the following pairs illustrates the two extreme examples of market structures? a. competition and oligopoly b. competition and monopoly c. monopoly and monopolistic competition d. oligopoly and monopolistic competition

B

QN=462 (17579) When a monopolistically competitive firm raises its price, a. quantity demanded falls to zero. b. quantity demanded declines but not to zero. c. the market supply curve shifts outward. d. quantity demanded remains constant.

A

QN=463 (17573) Refer to Table 16-2. Which industry has the highest concentration ratio? a. Industry A b. Industry B c. Industry C d. Industry D

B

QN=464 (17588) One characteristic of an oligopoly market structure is: a. firms in the industry are typically characterized by very diverse product lines. b. firms in the industry have some degree of market power. c. products typically sell at a price equal to their marginal cost of production. d. the actions of one seller have no impact on the profitability of other sellers.

D

QN=465 (17566) A monopolistically competitive firm is currently producing 10 units of output. At this level of output the firm is charging a price equal to $10, has marginal revenue equal to $6, has marginal cost equal to $6, and has average total cost equal to $12. From this information we can infer that a. (i) the firm is currently maximizing its profit. b. (ii) the profits of the firm are negative. c. (iii) firms are likely to leave this market in the long run. d. All of (i), (ii), and (iii) are correct.

B

QN=466 (17570) Which of these situations produces the largest profits for oligopolists? a. They reach a Nash equilibrium. b. They reach the monopoly outcome. c. They reach the competitive outcome. d. They produce a quantity of output that lies between the competitive outcome and the monopoly outcome.

D

QN=467 (17594) Because monopolistically competitive firms produce differentiated products, each firm a. faces a demand curve that is horizontal. b. faces a demand curve that is vertical. c. has no control over product price. d. has some control over product price.

B

QN=468 (17590) When a profit-maximizing firm in a monopolistically competitive market is in long-run equilibrium, a. the demand curve will be perfectly elastic. b. price exceeds or equals marginal cost. c. marginal cost must be falling. d. marginal revenue exceeds marginal cost.

C

QN=469 (17597) Which of the following conditions is characteristic of a monopolistically competitive firm in long-run equilibrium? a. P > demand and P = MR b. ATC > demand and MR = MC c. P > MC and demand = ATC d. P < ATC and demand > MR

B

QN=47 (17165) When economists make normative statements, they are a. speaking as scientists. b. speaking as policy advisers. c. making claims about how the world is. d. revealing that they are very liberal in their views of how the world works.

C

QN=470 (17576) Critics of advertising argue that advertising a. (i) creates desires that otherwise might not exist. b. (ii) hinders competition. c. (iii) often fails to convey substantive information. d. All of (i), (ii), and (iii) are correct.

C

QN=471 (17574) A monopolistically competitive firm chooses the quantity to produce where a. (i) price equals marginal cost. b. (ii) demand equals marginal cost. c. (iii) marginal revenue equals marginal cost. d. Both (i) and (iii) are correct.

D

QN=472 (17575) Monopolistic competition is characterized by (i) efficient scale (ii) markup pricing over marginal cost (iii) deadweight loss (iv) excess capacity a. (i) and (ii) only b. (ii) and (iv) only c. (i), (ii), and (iii) only d. (ii), (iii), and (iv) only

B

QN=473 (17596) Refer to Figure 16-4. Which of the graphs depicts a short-run equilibrium that will encourage the exit of some firms from a monopolistically competitive industry? a. panel (a) b. panel (b) c. panel (c) d. panel (d)

A

QN=474 (17581) When quality cannot be easily judged in advance, what provides consumers with information about the quality of a product? a. a brand name b. a tie-in c. the quantity available for sale d. the amount of deadweight loss

C

QN=475 (17577) A market is comprised of many firms as opposed to just one firm or a few firms a. only when it is perfectly competitive. b. only when it is perfectly competitive or oligopolistic. c. only when it is perfectly competitive or monopolistically competitive. d. when it is perfectly competitive, monopolistically competitive, or oligopolistic.

D

QN=488 (17624) In which of the following games is it clearly the case that the cooperative outcome of the game is good for the two players and good for society? a. Two guilty criminals have been captured by the police, and each prisoner decides whether to confess or to remain silent. b. Two airlines dominate air travel between City A and City B, and each airline decides whether to charge a "high" airfare or a "low" airfare. c. Two duopoly firms account for all of the production in a market, and each firm decides whether to produce a "high" amount of output or a "low" amount of output. d. Two oil companies own adjacent oil fields over a common pool of oil, and each company decides whether to drill one well or two wells.

A

QN=489 (17637) A central issue in the Microsoft antitrust lawsuit involved Microsoft's integration of its Internet browser into its Windows operating system, to be sold as one unit. This practice is known as a. tying. b. predation. c. wholesale maintenance. d. retail maintenance.

B

QN=49 (17148) Which of the following is not correct? a. Economists use some familiar words in specialized ways. b. Economics has its own language and its own way of thinking, but few other fields of study do. c. Supply, demand, elasticity, comparative advantage, consumer surplus, and deadweight loss are all terms that are part of the economist's language. d. The value of the economist's language lies in its ability to provide you with a new and useful way of thinking about the world in which you live.

B

QN=490 (17614) In which of the following games is it clearly the case that the cooperative outcome of the game is good for the two players and bad for society? a. Two oil companies own adjacent oil fields over a common pool of oil, and each company decides whether to drill one well or two wells. b. Two airlines dominate air travel between City A and City B, and each airline decides whether to charge a "high" airfare or a "low" airfare on flights between those two cities. c. Two superpowers decide whether to build new weapons or to disarm. d. In all of the above cases, the cooperative outcome of the game is good for the two players and bad for society

B

QN=491 (17625) The simplest type of oligopoly is a. monopoly. b. duopoly. c. monopolistic competition. d. oligopolistic competition.

A

QN=492 (17613) Figure 17-1. Two companies, ABC and XYZ, each decide whether to produce a high level of output or a low level of output. In the figure, the dollar amounts are payoffs and they represent annual profits for the two companies. Refer to Figure 17-1. The dominant strategy for ABC is to a. produce high output, and the dominant strategy for XYZ is to produce high output. b. produce high output, and the dominant strategy for XYZ is to produce low output. c. produce low output, and the dominant strategy for XYZ is to produce high output. d. produce low output, and the dominant strategy for XYZ is to produce low output.

A

QN=493 (17635) Refer to Table 17-15. In this game, a. neither player has a dominant strategy. b. both players have a dominant strategy. c. Firm A has a dominant strategy, but Firm B does not have a dominant strategy. d. Firm B has a dominant strategy, but Firm A does not have a dominant strategy.

D

QN=494 (17609) Which of the following would be most likely to contribute to the breakdown of a cartel in a natural resource (e.g., bauxite) market? a. high prices b. low price elasticity of demand c. high compatibility of member interests d. unequal member ownership of the natural resource

A

QN=495 (17619) Entry by new firms into a monopolistically competitive market a. (i) creates new consumer surplus. b. (ii) imposes a positive externality on existing firms. c. (iii) leads to the same externalities that are observed when new firms enter a perfectly competitive market. d. All of (i), (ii), and (iii) are correct.

C

QN=496 (17611) As a group, oligopolists would always earn the highest profit if they would a. produce the perfectly competitive quantity of output. b. produce more than the perfectly competitive quantity of output. c. charge the same price that a monopolist would charge if the market were a monopoly. d. operate according to their own individual self-interests.

A

QN=497 (17633) Refer to Table 17-2. Suppose there are exactly two sellers of gasoline in Pittsville: Exxoff and BQ. If Exxoff sells 300 gallons and BQ sells 400 gallons, then a. Exxoff's profit is $900 and BQ's profit is $1,200. b. Exxoff's profit is $2,100 and BQ's profit is $2,400. c. there is an excess demand for gasoline in Pittsville. d. there is an excess supply of gasoline in Pittsville.

A

QN=498 (17618) A profit-maximizing firm in a monopolistically competitive market is characterized by which of the following? a. Average revenue exceeds marginal revenue. b. Marginal revenue exceeds average revenue. c. Average revenue is equal to marginal revenue. d. Revenue is always maximized along with profit.

C

QN=499 (17627) An agreement among firms regarding price and/or production levels is called a. an antitrust market. b. a free-trade arrangement. c. collusion. d. a Nash agreement.

D

QN=5 (17126) Households and economies have each of the following in common EXCEPT both a. must allocate scarce resources. b. face many decisions. c. must allocate the goods and services they produce. d. must have a central decision maker.

C

QN=500 (17629) Game theory is important for the understanding of a. competitive markets. b. monopolies. c. oligopolies. d. all market structures.

B

QN=501 (17601) The theory of oligopoly provides another reason that free trade can benefit all countries because a. (i) increased competition leads to larger deadweight losses. b. (ii) as the number of firms within a given market increases, the price of the good decreases. c. (iii) as the number of firms within a given market increases, the profit of each firm increases. d. All of (i), (ii), and (iii) are correct.

D

QN=502 (17610) A special kind of imperfectly competitive market that has only two firms is called a. a two-tier competitive structure. b. an incidental monopoly. c. a doublet. d. a duopoly.

A

QN=529 (17665) If Emma's individual labor supply curve is upward sloping, then Emma responds to an increase in a. (i) the wage by working more hours per week. b. (ii) the opportunity cost of leisure by working fewer hours per week. c. (iii) the opportunity cost of leisure by taking more hours of leisure per week. d. Both (i) and (ii) are correct.

B

QN=503 (17600) Suppose that Sonny and Cher are duopolists in the music industry. In January, they agree to work together as a monopolist, charging the monopoly price for their music and producing the monopoly quantity of songs. By February, each singer is considering breaking the agreement. What would you expect to happen next? a. Sonny and Cher will determine that it is in each singer's best self interest to maintain the agreement. b. Sonny and Cher will each break the agreement. The new equilibrium quantity of songs will increase, and the new equilibrium price will decrease. c. Sonny and Cher will each break the agreement. The new equilibrium quantity of songs will decrease, and the new equilibrium price will increase. d. Sonny and Cher will each break the agreement. The new equilibrium quantity of songs will increase, and the new equilibrium price also will increase.

A

QN=504 (17630) Table 17-12. This table shows a game played between two players, A and B. The payoffs in the table are shown as (Payoff to A, Payoff to B). Refer to Table 17-12. Which of the following statements about this game is true? a. Up is a dominant strategy for A and Right is a dominant strategy for B. b. Up is a dominant strategy for A and Left is a dominant strategy for B. c. Down is a dominant strategy for A and Right is a dominant strategy for B. d. Down is a dominant strategy for A and Left is a dominant strategy for B.

C

QN=505 (17604) Predatory pricing involves a firm a. colluding with another firm to restrict output and raise prices. b. selling two individual products together for a single price rather than selling each product individually at separate prices. c. temporarily cutting the price of its product to drive a competitor out of the market. d. requiring that the firm reselling its product do so at a specified price.

C

QN=506 (17612) The likely outcome of the standard prisoners' dilemma game is that a. neither prisoner confesses. b. exactly one prisoner confesses. c. both prisoners confess. d. Not enough information is given to answer this question.

C

QN=507 (17631) An agreement between two duopolists to function as a monopolist usually breaks down because a. they cannot agree on the price that a monopolist would charge. b. they cannot agree on the output that a monopolist would produce. c. each duopolist wants a larger share of the market in order to capture more profit. d. each duopolist wants to charge a higher price than the monopoly price.

D

QN=508 (17622) After initial success, the OPEC cartel saw the price of oil and the revenues of its members decline due, in part, to a. the low elasticity of demand for oil in the short run. b. the large number of buyers from each member nation. c. surging demand for oil in the early 1980s. d. OPEC members failing to produce their agreed-upon production levels.

C

QN=509 (17608) Refer to Table 17-7. Suppose the market for this product is served by two duopolists who have formed a cartel and are colluding to set the price and quantity in this market. If the marginal cost to produce this product is constant at $2 per unit, then what price will the cartel set in this market? a. $4 b. $5 c. $6 d. $7

D

QN=51 (17157) Suppose an economy produces two goods, food and machines. This economy always operates on its production possibilities frontier. Last year, it produced 50 units of food and 30 machines. This year it experienced a technological advance in its machine-making industry. As a result, this year the society wants to produce 55 units of food and 30 machines. Which of the following statements is correct? a. Because the technological advance occurred in the machine-making industry, it will not be possible to increase food production without reducing machine production below 30. b. Because the technological advance occurred in the machine-making industry, increases in output can only occur in the machine industry. c. In order to increase food production in these circumstances without reducing machine production, the economy must reduce inefficiencies. d. The technological advance reduced the amount of resources needed to produce 30 machines, so these resources could be used to produce more food.

B

QN=510 (17615) In which of the following market structures is the number of sellers less than "many?" (i) monopolistic competition (ii) monopoly (iii) oligopoly a. (i) and (ii) only b. (ii) and (iii) only c. (ii) only d. In all of (i), (ii), and (iii).

C

QN=511 (17628) When firms are faced with making strategic choices in order to maximize profit, economists typically use a. the theory of monopoly to model their behavior. b. the theory of aggressive competition to model their behavior. c. game theory to model their behavior. d. cartel theory to model their behavior.

C

QN=512 (17636) The prisoners' dilemma game a. (i) is a situation in which two players both have dominant strategies which lead to the highest total payoff for the two players. b. (ii) has no Nash equilibrium since players, after agreeing to play their dominant strategy, will have an incentive to switch to another strategy. c. (iii) has a Nash equilibrium, but the Nash equilibrium outcome is not the outcome the players would agree to if they could cooperate with each other. d. Both (i) and (iii) are correct.

B

QN=513 (17602) The prisoners' dilemma is an important game to study because a. most games present zero-sum alternatives. b. it identifies the fundamental difficulty in maintaining cooperative agreements. c. strategic decisions faced by the prisoners are identical to those faced by firms engaged in competitive agreements. d. all interactions among firms are represented by this game.

A

QN=514 (17621) Table 17-19. The Chicken Game is named for a contest in which drivers test their courage by driving straight at each other. John and Paul have a common interest to avoid crashing into each other, but they also have a personal, competing interest to not turn first to demonstrate their courage to those observing the contest. The payoff table for this situation is provided below. The payoffs are shown as (John, Paul). Refer to Table 17-19. If Paul chooses Drive Straight, what will John choose to do and what will John's payoff equal? a. Turn, 5 b. Drive Straight, 0 c. Turn, 20 d. Drive Straight, 5

A

QN=53 (17172) Factors of production are a. used to produce goods and services. b. also called output. c. abundant in most economies. d. assumed to be owned by firms in the circular-flow diagram.

D

QN=550 (17678) Refer to Figure 18-6. The shift of the labor demand curve from D1 to D2 could possibly be explained by a. (i) technological progress. b. (ii) an increase in the price of firms' output. c. (iii) an increase in the supply of a relevant factor of production other than labor. d. All of (i), (ii), and (iii) are correct.

B

QN=515 (17620) Table 17-9 Two cigarette manufacturers (Firm A and Firm B) are faced with lawsuits from states to recover the healthcare related expenses associated with cigarette smoking. Both cigarette firms have evidence that indicates that cigarette smoke causes lung cancer (and other related illnesses). State prosecutors do not have access to the same data used by cigarette manufacturers and thus will have difficulty recovering full costs without the help of at least one cigarette firm study. Each firm has been presented with an opportunity to lower its liability in the suit if it cooperates with attorneys representing the states. Refer to Table 17-9. When this game reaches a Nash equilibrium, profits for Firm A and Firm B will be a. $-5 and $-50, respectively. b. $-10 and $-10, respectively. c. $-20 and $-15, respectively. d. $-50 and $-5, respectively.

A

QN=516 (17616) Firm A produces and sells in a market that is characterized by highly differentiated consumer goods. Firm B produces and sells industrial products. Firm C produces and sells an agricultural commodity. Which firm is likely to spend the greatest portion of its total revenue on advertising? a. Firm A b. Firm B c. Firm C d. There is no reason to believe that any one of the three firms would spend a greater portion of its total revenue on advertising than the other two firms.

D

QN=517 (17617) Which of the following statements is correct? All theoretical arguments point to the fact that advertising leads to a. (i) higher prices and less competitive markets. b. (ii) higher prices and more competitive markets. c. (iii) lower prices and less competitive markets. d. None of (i), (ii), and (iii) are correct; theoretical arguments fail to resolve the question of advertising's effect on prices and competition.

C

QN=518 (17605) In which of the following markets are strategic interactions among firms most likely to occur? a. the market for bottled water b. the market for piano lessons c. the market for tennis rackets d. the market for corn

C

QN=519 (17632) If a certain market were a monopoly, then the monopolist would maximize its profit by producing 1,000 units of output. If, instead, that market were a duopoly, then which of the following outcomes would be most likely if the duopolists successfully collude? a. Each duopolist produces 1,000 units of output. b. Each duopolist produces 600 units of output. c. One duopolist produces 400 units of output and the other produces 600 units of output. d. One duopolist produces 800 units of output and the other produces 400 units of output.

A

QN=52 (17167) Refer to Figure 2-7. In order to reach point C, the economy would have to a. (i) acquire more resources or experience a technological advance. b. (ii) begin using its available resources more efficiently than it is currently using them. c. (iii) shift resources away from the production of ribeye steaks and toward production of books. d. None of (i), (ii), and (iii) are correct; the economy will never be able to reach point C.

A

QN=520 (17603) The players in a two-person game are choosing between Strategy X and Strategy Y. If the second player chooses Strategy X, the first player's best outcome is to select X. If the second player chooses Strategy Y, the first player's best outcome is to select X. For the first player, Strategy X is called a a. dominant strategy. b. collusive strategy. c. repeated-trial strategy. d. cartel strategy.

D

QN=521 (17634) As the number of sellers in an oligopoly becomes very large, a. (i) the quantity of output approaches the socially efficient quantity. b. (ii) the price approaches marginal cost. c. (iii) the price effect is diminished. d. All of (i), (ii), and (iii) are correct.

C

QN=522 (17626) Cartels are difficult to maintain because a. antitrust laws are difficult to enforce. b. cartel agreements are conducive to monopoly outcomes. c. there is always tension between cooperation and self-interest in a cartel. d. firms pay little attention to the decisions made by other firms.

B

QN=523 (17623) The practice of selling a product to retailers and requiring the retailers to charge a specific price for the product is called a. fixed retail pricing. b. resale price maintenance. c. cost plus pricing. d. unfair trade.

B

QN=524 (17606) The equilibrium quantity in markets characterized by oligopoly is a. higher than in monopoly markets and higher than in perfectly competitive markets. b. higher than in monopoly markets and lower than in perfectly competitive markets. c. lower than in monopoly markets and higher than in perfectly competitive markets. d. lower than in monopoly markets and lower than in perfectly competitive markets.

D

QN=525 (17607) Which of the following statements is correct? a. (i) If duopolists successfully collude, then their combined output will be equal to the output that would be observed if the market were a monopoly. b. (ii) Although the logic of self-interest decreases a duopoly's price below the monopoly price, it does not push the duopolists to reach the competitive price. c. (iii) Although the logic of self-interest increases a duopoly's level of output above the monopoly level, it does not push the duopolists to reach the competitive level. d. All of (i), (ii), and (iii) are correct.

A

QN=526 (17659) Suppose that a large tornado destroys the fleet of fire trucks for the city of Omaha, Nebraska. What happens to the earnings of firefighters in Omaha? a. The reduction in the supply of fire trucks reduces the marginal productivities of Omaha firefighters, which causes the equilibrium wage to fall. b. The reduction in the supply of fire trucks increases the marginal productivities of Omaha firefighters, which causes the equilibrium wage to fall. c. The reduction in the supply of fire trucks reduces the marginal productivities of Omaha firefighters, which causes the equilibrium wage to rise. d. The reduction in the supply of fire trucks increases the marginal productivities of Omaha firefighters, which causes the equilibrium wage to rise.

C

QN=527 (17674) Dave is the owner of Dave's Pizza Palace. Dave is a profit-maximizing owner whose firm operates in a competitive market. An additional worker costs Dave $200 and has a marginal productivity of 40 pizzas. Assuming no other variable costs, what is the marginal cost of a pizza? a. $200 b. $8 c. $5 d. There is insufficient information available to answer this question.

B

QN=528 (17671) 5. Refer to Table 18-3. This table describes the number of baseballs a manufacturer can produce per day with different quantities of labor. Each baseball sells for $5 in a competitive market. For which level of employment is the marginal product of labor greatest? a. 1 worker b. 2 workers c. 3 workers d. 4 workers

A

QN=530 (17651) What happens to the labor supply curve for academic economists when the wages paid to government economists increase? a. The labor supply curve for academic economists shifts to the left. b. The labor supply curve for academic economists shifts to the right. c. The labor supply curve for academic economists will become backward-sloping. d. The labor supply curve for academic economists will not change.

B

QN=531 (17670) If the demand curve for computer games shifts to the left, then the value of the marginal product of labor for computer game authors will a. rise. b. fall. c. remain unchanged. d. rise or fall; either is possible.

D

QN=532 (17668) The term "factor market" applies to the market for a. (i) labor. b. (ii) capital. c. (iii) land. d. All of (i), (ii), and (iii) are correct.

A

QN=533 (17657) 1. Refer to Figure 18-1. Suppose the firm hires each unit of labor for $700 per week, and each unit of output sells for $9. How many workers will the firm hire to maximize its profit? a. 2 b. 3 c. 4 d. 5

B

QN=534 (17644) An increase in the supply of labor has the effect of decreasing the a. (i) wage. b. (ii) marginal product of labor. c. (iii) value of the marginal product of labor. d. All of (i), (ii), and (iii) are correct.

B

QN=535 (17676) Suppose that technological progress increases the productivity of teachers. Which of the following accurately describes the labor market for teachers after the technological change? a. Wages will rise and quantity of teachers employed will fall. b. Wages will rise and the quantity of teachers employed will rise. c. Wages will fall and the quantity of teachers employed will fall. d. Wages will fall and the quantity of teachers employed will rise.

C

QN=536 (17656) The distinction between purchase price and rental price applies to which factor(s) of production? a. land only b. capital only c. land and capital only d. land, capital, and labor

D

QN=537 (17647) Since workers in the U.S. economy receive most of the total income earned, which of the following factors of production is considered to be the most important? a. Profit b. Wages c. Interest d. Labor

B

QN=538 (17667) The factors of production are best defined as the a. output produced from raw materials. b. inputs used to produce goods and services. c. wages paid to the workforce. d. goods and services sold in the market.

B

QN=539 (17654) Which of the following best illustrates the concept of "derived demand?" a. An increase in the wages of autoworkers will lead to an increase in the demand for robots in automobile factories. b. An automobile producer's decision to supply more cars will lead to an increase in the demand for automobile production workers. c. An automobile producer's decision to supply more minivans results from a decrease in the demand for station wagons. d. An increase in the price of gasoline will lead to an increase in the demand for small cars.

D

QN=54 (17155) The two basic reasons why economists often appear to give conflicting advice to policymakers are differences in a. opinions and education. b. opinions and values. c. scientific judgments and education. d. scientific judgments and values.

D

QN=540 (17662) Capital, labor, and land a. (i) have derived demands. b. (ii) are factors of production. c. (iii) are inputs used in the production of goods and services. d. All of (i), (ii), and (iii) are correct.

C

QN=542 (17664) Competitive firms decide how much output to sell by producing output until the price of the good equals a. marginal product. b. the value of marginal product. c. marginal cost. d. marginal profit.

D

QN=543 (17672) When a firm maximizes profit, a. it will hire workers up to the point where the marginal product of labor is equal to the product price. b. it will hire workers up to the point where the marginal product of labor is equal to the wage. c. it will hire workers up to the point where the value of the marginal product of labor is equal to the product price. d. it will hire workers up to the point where the value of the marginal product of labor is equal to the wage.

B

QN=544 (17669) If the price of airline tickets falls, what will happen to the demand curve for flight attendants? a. It will shift to the right. b. It will shift to the left. c. The direction of the shift is ambiguous. d. It will remain unchanged.

A

QN=545 (17663) Suppose that a competitive firm hires labor up to the point at which the value of the marginal product equals the wage. If the firm pays a wage of $700 per week and the marginal product of labor equals 20 units per week, then the marginal cost of producing an additional unit of output is a. $35 b. $70 c. $700 d. We do not have enough information to answer this question.

A

QN=546 (17652) Which of the following best describes the economy's stock of equipment and structures? a. capital b. aggregate demand c. long-term inventory d. aggregate stock

D

QN=547 (17653) Labor markets are different from most other markets because labor demand is a. represented by a vertical line on a supply-demand diagram. b. represented by an upward-sloping line on a supply-demand diagram. c. such an elusive concept. d. derived.

D

QN=548 (17660) 4. Refer to Figure 18-2. The value-of-marginal-product curve that is drawn could be relabeled as the firm's a. production function. b. total revenue curve. c. labor supply curve. d. labor demand curve.

A

QN=549 (17661) Consider the market for university economics professors. Suppose the opportunity cost of going to graduate school to get a Ph.D. in economics decreases for many individuals. Since it generally takes about five years to get a Ph.D. in economics, holding all else constant, what will likely happen to the equilibrium quantity of university economics professors in five years? a. The equilibrium quantity will increase. b. The equilibrium quantity will decrease. c. The equilibrium quantity will not change. d. It is not possible to determine what will happen to the equilibrium quantity.

D

QN=55 (17183) A model can be accurately described as a a. theoretical abstraction with very little value. b. device that is useful only to the people who created it. c. realistic and carefully constructed theory. d. simplification of reality.

C

QN=551 (17655) Which of the following statements is correct? An individual worker's labor supply curve a. can never slope downward. b. slopes downward if that person responds to a higher wage by taking fewer hours of leisure per week. c. slopes downward if that person responds to a higher opportunity cost of leisure by working fewer hours per week. d. is horizontal if that person works the same number of hours per week, regardless of the opportunity cost of leisure.

A

QN=552 (17658) Scenario 18-3 Jerry has two jobs, one for the winter and one for the summer. In the winter, he works as a lift attendant at a ski resort where he earns $10 per hour. During the summer, he drives a tour bus around the ski resort, earning $12 per hour. Refer to Scenario 18-3. During the summer months, what is Jerry's opportunity cost of taking an hour off work to go hiking? a. $12 b. between $10 and $12 c. $10 d. less than $10

A

QN=553 (17677) Consider the labor market for heath care workers. Because of the aging population in the United States, the output price for health care services has increased. Holding all else equal, what effect does this have on the labor market for health care employees? a. The equilibrium wage increases and the equilibrium quantity of labor increases. b. The equilibrium wage increases and the equilibrium quantity of labor decreases. c. The equilibrium wage decreases and the equilibrium quantity of labor increases. d. The equilibrium wage decreases and the equilibrium quantity of labor decreases.

B

QN=554 (17640) We observe a profit-maximizing firm hiring its 51st employee. It is possible to infer that, when 50 employees are hired, the a. wage exceeds the value of the marginal product of labor. b. value of the marginal product of labor exceeds the wage. c. marginal product of labor is increasing. d. firm is attempting to increase its market share.

B

QN=555 (17673) John owns a number of hot dog stands in New York City. He hires workers to sell hot dogs at his stands. Which of the following events will lead to a decrease in John's demand for hot dog vendors? a. Hollywood glamorization of a new movie about a hot dog vendor leads hundreds of high-school students in New York City to apply for a job at John's. b. The price of hot dogs falls. c. The local hot dog vendors form a union increasing hot dog vendor wages. d. The demand curve for hot dogs shifts to the right.

C

QN=556 (17649) Refer to Table 18-2. Suppose this firm charges a price of $5 per unit of output and pays workers a wage equal to $160 per day. How many workers should this firm hire to maximize its profit? a. 2 workers b. 3 workers c. 4 workers d. 5 workers

D

QN=557 (17666) The demand curve for capital a. is vertical. b. is horizontal. c. is derived from households' decisions concerning saving and spending. d. reflects the marginal productivity of capital.

C

QN=558 (17645) In economics, the term capital is used to refer to a. (i) money. b. (ii) stocks and bonds. c. (iii) equipment and structures used in production. d. All of (i), (ii), and (iii) are correct.

C

QN=559 (17646) A decrease in population can be expected to a. increase the marginal product of land. b. decrease the supply of land. c. decrease the rents on land. d. increase the demand for land.

A

QN=56 (17185) Refer to Table 2-2. What is the opportunity cost to Batterland of increasing the production of pancakes from 150 to 300? a. 75 waffles b. 150 waffles c. 250 waffles d. 325 waffles

D

QN=560 (17643) For a competitive firm experiencing diminishing marginal productivity, the value of the marginal product (i) increases when the price of output decreases. (ii) changes when marginal product changes. (iii) diminishes as the number of workers rises. a. (i) and (ii) b. (i) and (iii) c. (ii) and (iii) d. All of (i), (ii), and (iii) are correct.

C

QN=561 (17638) 1. Refer to Table 18-5. What is the fourth worker's marginal revenue product? a. $100 b. $40 c. $400 d. $500

B

QN=562 (17639) If a firm experiences diminishing marginal productivity of labor, the marginal product a. increases as total product increases. b. decreases as total product increases. c. increases as total product decreases. d. decreases as total product decreases.

B

QN=563 (17648) The following table shows the production function for a particular business. The numbers represent the various labor and output combinations the firm may choose for its output on a daily basis. Refer to Table 18-2. What is the marginal product of the third unit of labor? a. 40 units b. 50 units c. 60 units d. 180 units

B

QN=564 (17675) Which of the following events would lead to an increase in the supply of labor? a. (i) The price of a firm's product increases. b. (ii) A country experiences an increase in immigrant labor. c. (iii) The development of a new labor-augmenting technology. d. All of (i), (ii), and (iii) are correct.

B

QN=565 (17642) When economists refer to a firm's capital, they are likely to be using the term to describe the a. markets for final goods and services. b. stock of equipment and buildings used in production. c. amount of bank financing used by the firm. d. amount of financing provided by the equity markets.

A

QN=566 (17650) Which of the following would shift a market labor supply curve to the left? a. an increase in the wage paid to workers in a competing market b. labor-augmenting technology c. a change in worker tastes so that workers want to retire later d. a decrease in the supply of other factors such as capital

C

QN=567 (17679) A consumer has preferences over two goods: books and movies. The three bundles shown in the table below lie on the same indifference curve for the consumer. Which of the following properties of indifference curves would this consumer's preferences violate? a. Indifference curves are downward sloping. b. Indifference curves do not cross. c. Indifference curves are bowed inward. d. These bundles do not violate any of the properties of indifference curves.

B

QN=568 (17705) Refer to Figure 21-9. Bundle C represents a point where a. MRSxy > Py/Px. b. MRSxy = Px/Py. c. MRSxy < Px/Py. d. MRSxy > Px/Py.

C

QN=569 (17688) A budget constraint illustrates the a. prices that a consumer chooses to pay for products he consumes. b. purchases made by consumers. c. consumption bundles that a consumer can afford. d. consumption bundles that give a consumer equal satisfaction.

B

QN=576 (17715) Bundle L contains 10 units of good X and 20 units of good Y. Bundle M contains 8 units of good X and 21 units of good Y. The consumer is indifferent between bundle L and bundle M. Assume that the consumer's preferences satisfy the four properties of indifference curves. Which of the following correctly expresses the marginal rate of substitution of good X for good Y between these two points? a. The consumer will give up 1 unit of good X to gain 2 units of good Y. b. The consumer will give up 2 units of good X to gain 1 unit of good Y. c. The price of good X is twice as large as the price of good Y. d. The price of good X is half as large as the price of good Y.

B

QN=577 (17682) Suppose at the consumer's current consumption bundle the marginal rate of substitution of cheese for wine is 1/2 bottle of wine per pound of cheese. The price of one pound of cheese is $6, and the price of a bottle of wine is $10. The consumer should increase his consumption of a. cheese, decrease his consumption of wine, and move to a lower indifference curve. b. cheese, decrease his consumption of wine, and move to a higher indifference curve. c. wine, decrease consumption of cheese, and move to a higher indifference curve. d. cheese, decrease consumption of wine, and remain on the same indifference curve.

C

QN=578 (17681) An optimizing consumer will select a consumption bundle in which a. income is maximized, and prices are minimized. b. utility is maximized, and prices are minimized. c. utility is maximized, subject to budget constraints. d. utility is maximized, and indifference curves are linear.

D

QN=579 (17697) A consumer's preferences provide a a. ranking of the set of bundles that happen to fall on indifference curves. b. relative ranking of bundles that provide more of all goods. c. framework for evaluating market equilibriums. d. complete ranking of all possible consumption bundles.

A

QN=58 (17163) Refer to Figure 2-9, Panel (a). In order to gain 2 radios by moving from point S to point T, society must sacrifice a. (i) 6 televisions. b. (ii) employment. c. (iii) efficiency. d. More than one of (i), (ii), and (iii) is correct.

D

QN=580 (17717) Giffen goods have positively-sloped demand curves because they are a. inferior goods with no substitution effect. b. normal goods with no substitution effect. c. inferior goods for which the substitution effect outweighs the income effect. d. inferior goods for which the income effect outweighs the substitution effect.

A

QN=581 (17707) The slope of the budget constraint is determined by the a. relative price of the goods measured on the axes. b. relative price of the goods measured on the axes and the consumer's income. c. endowment of productive resources. d. preferences of the consumer.

D

QN=582 (17694) When the price of pizza falls, the substitution effect, for normal goods Pepsi and pizza, causes a a. shift to a lower indifference curve so the consumer buys more Pepsi. b. shift to a higher indifference curve so the consumer buys more Pepsi. c. movement along the indifference curve so the consumer buys more Pepsi. d. movement along the indifference curve so the consumer buys less Pepsi.

A

QN=583 (17714) Indifference curves tend to be bowed inward because of diminishing a. (i) marginal rates of substitution. b. (ii) demand for the good as prices rise. c. (iii) income. d. Both (i) and (ii) are correct.

D

QN=584 (17712) The following diagram shows two budget lines: A and B. Which of the following could explain the change in the budget line from A to B? a. a decrease in income and a decrease in the price of X b. a decrease in income and an increase in the price of X c. an increase in income and a decrease in the price of X d. an increase in income and an increase in the price of X

B

QN=585 (17701) Suppose a consumer spends his income on CDs and DVDs. If his income decreases, the budget constraint for CDs and DVDs will a. shift outward, parallel to the original budget constraint. b. shift inward, parallel to the original budget constraint. c. rotate outward along the CD axis because he can afford more CDs. d. rotate outward along the DVD axis because he can afford more DVDs.

D

QN=586 (17685) 2. Refer to Figure 21-2. Which of the following statements is not correct? a. Points W, X, and Y all cost the consumer the same amount of money. b. Point Z is unaffordable for the consumer given his budget constraint. c. Point V costs less than point Z. d. Points W, X, and Y give the consumer the same level of satisfaction.

D

QN=587 (17699) Giffen goods have positively-sloped demand curves because they are a. inferior goods with no substitution effect. b. normal goods with no substitution effect. c. inferior goods for which the substitution effect outweighs the income effect. d. inferior goods for which the income effect outweighs the substitution effect.

C

QN=588 (17698) Wilbur consumes two goods, bacon and eggs. He has maximized his utility given his income. Eggs costs $2 per dozen, and he consumes them to the point where the marginal utility he receives is 6. Bacon cost $4 per serving, and the relationship between the marginal utility that Wilbur gets from eating bacon and the number of servings he eats per month is as follows: How many servings of bacon does Wilbur buy each month? a. 1 b. 2 c. 3 d. 4

D

QN=589 (17692) 2. Refer to Figure 21-02. Which of the graphs in the figure reflects an increase in consumer's income? a. graph (a) b. graph (b) c. graph (c) d. graph (d)

D

QN=59 (17176) The field of economics is traditionally divided into two broad subfields, a. national economics and international economics. b. consumer economics and producer economics. c. private sector economics and public sector economics. d. microeconomics and macroeconomics.

A

QN=590 (17686) Jake faces tradeoffs between consuming in the current period when he is young and consuming in a future period when he is old. Jake experiences a decrease in the current interest rate he earns on his savings. Jake will save a. less in the current period if the substitution effect is greater than the income effect. b. less in the current period if the income effect is greater than the substitution effect. c. more in the current period if the substitution effect is greater than the income effect. d. more in the current period, regardless of the sizes of the income and substitution effects.

C

QN=599 (17711) Suppose a consumer has an income of $800 per month and that she spends her entire income each month on beer and sausage. The price of a pint of beer is $5, and the price of a sausage is $4. Which of the following combinations of beers and bratwursts represents a point that would lie directly on the consumer's budget constraint? a. 160 beers and 200 sausages b. 40 beers and 50 sausages c. 80 beers and 100 sausages d. 80 beers and 0 sausages

B

QN=6 (17130) An increase in the overall level of prices in an economy is referred to as a. economic growth. b. inflation. c. monetary policy. d. supply shocks.

A

QN=60 (17159) Economists use models in order to a. learn how the economy works. b. make their profession appear more precise. c. make economics difficult for students. d. make sure that all of the details of the economy are included in their analysis.

B

QN=600 (17703) 3. Refer to Figure 21-7. Which of the following statements is correct? a. Bundle A is preferred equally to bundle E. b. Bundle A is preferred equally to bundle C. c. Bundle B contains more cake than bundle C. d. The bundles along indifference curve Indifference Curve 2 are preferred to those along indifference curve Indifference Curve 3.

C

QN=601 (17713) A family on a trip budgets $800 for meals and hotel accommodations. Suppose the price of a meal is $40. In addition, suppose the family could afford a total of 8 nights in a hotel if they don't buy any meals. How many meals could the family afford if they gave up two nights in the hotel? a. 1 b. 2 c. 5 d. 8

D

QN=602 (17700) Suppose that you have $100 today and expect to receive $100 one year from today. Your money market account pays an annual interest rate of 25%, and you may borrow money at that interest rate. If you save all your money, how much money will you have one year from today? a. $100 b. $125 c. $200 d. $225

D

QN=603 (17704) When a consumer is purchasing the best combination of two goods, X and Y, subject to a budget constraint, we say that the consumer is at an optimal choice point. A graph of an optimal choice point shows that it occurs a. (i) along the highest attainable indifference curve. b. (ii) where the indifference curve is tangent to the budget constraint. c. (iii) where the marginal utility per dollar spent is the same for both X and Y. d. All of (i), (ii), and (iii) are correct.

A

QN=604 (17696) An increase in income will cause a consumer's budget constraint to a. shift outward, parallel to its initial position. b. shift inward, parallel to its initial position. c. pivot around the horizontal axis. d. pivot around the vertical axis.

C

QN=605 (17680) The marginal rate of substitution is a. the slope of a budget constraint. b. always constant. c. the slope of an indifference curve. d. the point at which the budget constraint and the indifference curve are tangent.

C

QN=61 (17154) Positive statements are a. prescriptive. b. claims about how the world should be. c. claims about how the world is. d. made by economists speaking as policy advisers.

C

QN=62 (17181) Which of the following is correct? a. A horizontal line has an infinite slope, and a vertical line has a zero slope. b. A horizontal line has a slope of 1, and a vertical line has a slope of -1. c. A horizontal line has a zero slope, and a vertical line has an infinite slope. d. A horizontal line has a slope of -1, and a vertical line has a slope of 1.

B

QN=63 (17179) In the circular-flow diagram, which of the following items flows from households to firms through the markets for the factors of production? a. goods and services b. land, labor, and capital c. dollars spent on goods and services d. wages, rent, and profit

C

QN=64 (17169) The production possibilities frontier is a graph that shows the various combinations of output that an economy a. should produce. b. wants to produce. c. can produce. d. demands.

D

QN=65 (17171) In the circular-flow diagram, firms produce a. (i) goods and services using factors of production. b. (ii) output using inputs. c. (iii) factors of production using goods and services. d. Both (i) and (ii) are correct.

C

QN=66 (17178) Economists build economic models by a. generating data. b. conducting controlled experiments in a lab. c. making assumptions. d. reviewing statistical forecasts.

B

QN=67 (17168) In the circular-flow diagram, a. (i) firms own the factors of production. b. (ii) the factors of production are labor, land, and capital. c. (iii) the factors of production are also called "output." d. All of (i), (ii), and (iii) are correct.

B

QN=68 (17161) Refer to Figure 2-2. Boxes C and D represent a. households and firms. b. the goods and services market and the factors of production market. c. the goods and services market and the financial market. d. households and government.

C

QN=69 (17150) A demand curve shows the relationship a. between income and quantity demanded. b. between price and income. c. between price and quantity demanded. d. among income, price, and quantity demanded.

A

QN=7 (17144) In an economy in which decisions are guided by prices and individual self-interest, there is a. the potential to achieve efficiency in production. b. a strong need for government intervention in the market. c. less efficiency than would be observed in a centrally-planned economy. d. more need for a strong legal system to control individual greed than would be needed in a centrally-planned economy.

B

QN=70 (17151) Refer to Figure 2-2. Alisha regularly buys fruits and vegetables at a grocery store. Santo regularly pays a lawn-care company to mow his lawn. If the flow of fruits and vegetables from the grocery store to Alisha is represented by an arrow from Box C to Box B of this circular-flow diagram, then the money paid by Santo to the lawn-care company is represented by an arrow a. from Box A to Box D. b. from Box B to Box C. c. from Box C to Box B. d. from Box D to Box A.

B

QN=71 (17180) The slope of a line is equal to a. the change in the value of x divided by the change in the value of y. b. the change in the value of y divided by the change in the value of x. c. the horizontal distance divided by the vertical distance. d. the value of y divided by the value of x.

C

QN=83 (17213) Which of the following events would cause the price of oranges to fall? a. (i) There is a shortage of oranges. b. (ii) An article is published in which it is claimed that tangerines cause a serious disease, and oranges and tangerines are substitutes. c. (iii) The price of land throughout Florida decreases, and Florida produces a significant proportion of the nation's oranges. d. All of (i), (ii), and (iii) are correct.

B

QN=84 (17199) The law of demand states that, other things equal, a. an increase in price causes quantity demanded to increase. b. an increase in price causes quantity demanded to decrease. c. an increase in quantity demanded causes price to increase. d. an increase in quantity demanded causes price to decrease.

A

QN=85 (17186) In a market economy, supply and demand determine a. both the quantity of each good produced and the price at which it is sold. b. the quantity of each good produced, but not the price at which it is sold. c. the price at which each good is sold, but not the quantity of each good produced. d. neither the quantity of each good produced nor the price at which it is sold.

D

QN=86 (17191) In a market economy, a. supply determines demand and demand, in turn, determines prices. b. demand determines supply and supply, in turn, determines prices. c. the allocation of scarce resources determines prices and prices, in turn, determine supply and demand. d. supply and demand determine prices and prices, in turn, allocate the economy's scarce resources.

A

QN=87 (17214) Two goods are substitutes when a decrease in the price of one good a. decreases the demand for the other good. b. decreases the quantity demanded of the other good. c. increases the demand for the other good. d. increases the quantity demanded of the other good.

C

QN=88 (17189) A movement downward and to the left along a supply curve is called a. an increase in supply. b. a decrease in supply. c. a decrease in quantity supplied. d. an increase in quantity supplied.

C

QN=89 (17192) Which of these statements best represents the law of demand? a. When buyers' tastes for a good increase, they purchase more of the good. b. When income levels increase, buyers purchase more of most goods. c. When the price of a good decreases, buyers purchase more of the good. d. When buyers' demands for a good increase, the price of the good increases.

A

QN=9 (17131) Moira decides to spend two hours taking a nap rather than attending her classes. Her opportunity cost of napping is a. the value of the knowledge she would have received had she attended class. b. the $30 she could have earned if she had worked at her job for those two hours. c. the value of her nap less the value of attending class. d. nothing, since she would valued sleep more than attendance at class.

C

QN=90 (17193) Which of the following is not held constant in a demand schedule? a. Income b. Tastes c. Price d. Expectations

D

QN=91 (17215) Two goods are complements when a decrease in the price of one good a. decreases the quantity demanded of the other good. b. decreases the demand for the other good. c. increases the quantity demanded of the other good. d. increases the demand for the other good.

B

QN=92 (17222) What will happen in the rice market now if sellers expect higher rice prices in the near future? a. The supply of rice will increase. b. The supply of rice will decrease. c. The supply of rice will be unaffected. d. The demand for rice will decrease.

C

QN=93 (17194) Other things equal, when the price of a good falls, the a. quantity demanded of the good decreases. b. supply decreases. c. quantity supplied of the good decreases. d. demand increases.

A

QN=94 (17196) In a market economy, supply and demand determine a. both the quantity of each good produced and the price at which it is sold. b. the quantity of each good produced, but not the price at which it is sold. c. the price at which each good is sold, but not the quantity of each good produced. d. neither the quantity of each good produced nor the price at which it is sold.

C

QN=95 (17210) The forces that make market economies work are a. work and leisure. b. politics and religion. c. supply and demand. d. taxes and government spending.

D

QN=96 (17218) An increase in the price of rubber coincides with an advance in the technology of tire production. As a result of these two events, a. (i) the demand for tires decreases and the supply of tires increases. b. (ii) the demand for tires is unaffected and the supply of tires decreases. c. (iii) the demand for tires is unaffected and the supply of tires increases. d. None of (i), (ii), and (iii) is necessarily correct.

C

QN=97 (17198) In a competitive market, the quantity of a product produced and the price of the product are determined by a. (i) buyers. b. (ii) sellers. c. (iii) both buyers and sellers. d. None of (i), (ii), and (iii) is correct.

C

QN=98 (17190) Which of the following would most likely serve as an example of a monopoly? a. a bakery in a large city b. a bank in a large city c. a local cable television company d. a small group of corn farmers

D

QN=99 (17200) In a free market, who determines how much of a good will be sold and the price at which it is sold? a. suppliers b. demanders c. the government d. suppliers and demanders together

C

Refer to Figure 18-4. What is measured along the vertical axis on the graph? a. The quantity of automobiles produced b. The price of automobiles c. The wage paid to automobile workers d. Time spent by workers producing automobiles

C

The minimum wage was instituted to ensure workers a. a middle-class standard of living. b. employment. c. a minimally adequate standard of living. d. unemployment compensation.


Ensembles d'études connexes

F342: Cost Accounting Chapter 3-5

View Set

Chapter 12 Skin, Hair, and Nails

View Set

VNSG 1304: Ch. 3 Prep U Questions

View Set

Chapter 13- Emergency Situations

View Set

Psyc chapter 8, Chapter 6, Psyc Chapter 5, Chapter 9 Motivation and emotion, Chapter 11: Personality Theory and Assessment, Chapter 10 Health and Stress, 12, 13

View Set

Chemistry Different types of solution

View Set

Cardiovascular system study guide

View Set